0% found this document useful (0 votes)
229 views85 pages

Medical Test Insights for PG Aspirants

Uploaded by

Kailash Khatri
Copyright
© © All Rights Reserved
We take content rights seriously. If you suspect this is your content, claim it here.
Available Formats
Download as PDF, TXT or read online on Scribd
0% found this document useful (0 votes)
229 views85 pages

Medical Test Insights for PG Aspirants

Uploaded by

Kailash Khatri
Copyright
© © All Rights Reserved
We take content rights seriously. If you suspect this is your content, claim it here.
Available Formats
Download as PDF, TXT or read online on Scribd

PARADISE WEEKLY MODEL TEST – CEE MD/MS BASED QAE SHRAWAN 20

PARADISE WEEKLY MODEL TEST


CEE MD/MS BASED MODEL TEST
(Saturday, Shrawan 20, 2080)
SCAN THE QR CODE AND JOIN THE COMMUNITY OF PG ASPIRANTS IN VIBER
- Discuss with fellow aspirants.
- Get notices about MD/MS Entrances
- Get notices about mock tests, results and offers
- Ask confusing questions to the experts
- Scroll the groups, and learn.

Questions Answers Explanations (QAE)

Medicine

1. A 30-year-old quadriplegic male presents to ER with fever, dyspnea, and a cough. He has a chronic
indwelling Foley catheter. Recurrent urinary tract infections have been a problem for a number of years.
He has been on therapy to suppress the urinary tract infections. On examination, mild wheezing is audible
over both lungs. A diffuse erythematous rash is noted. The chest x-ray shows a diffuse alveolar infiltrate.
Which of the following is the most likely diagnosis?
a. Sepsis with ARDS secondary to UTI
b. Healthcare-related pneumonia
c. A drug reaction to one of his medications
d. Lymphocytic interstitial pneumonitis

Ans: c
Ref: Harrison’s Principles of Internal Medicine; 19th Edition, Page No: 1686
• The clues to this diagnosis are recurrent urinary tract infections and the use of suppressive therapy to
control these infections. Nitrofurantoin is commonly used for this purpose. Nitrofurantoin can cause an
acute hypersensitivity pneumonitis. This condition can progress to a chronic alveolitis with pulmonary
fibrosis. The presenting symptoms are fever, chills, cough, and bronchospasm.
• In addition, the patient may experience arthralgias, myalgias, and an erythematous rash. The chest x-ray
will show interstitial or alveolar infiltrates. CBC often shows leukocytosis with a high percentage of
eosinophils. The treatment is to discontinue the nitrofurantoin, and begin corticosteroids.
• Sepsis secondary to a urinary tract infection, and a healthcare-related pneumonia might be considered.
However, these would not present with a diffuse erythroderma or eosinophilia. Acute bacterial infections
cause a neutrophilic leukocytosis; eosinophils are usually undetectable owing to the stress effect of
catecholamines and cortisol

Paradise Institute & Self Study Centre Page 1


PARADISE WEEKLY MODEL TEST – CEE MD/MS BASED QAE SHRAWAN 20

2. A 50-year-old woman complains of pain and swelling in her proximal interphalangeal joints, both wrists,
and both knees. She complains of morning stiffness. She had a hysterectomy 10 years ago. Physical
examination shows swelling and synovial thickening of the proximal interphalangeal (PIP) joints.
Hemoglobin is 10.3 g/dL, MCV is 80 fL, serum iron is 28 μg/dL, iron-binding capacity is 200 μg/dL
(normal 250-370 μg/dL), and saturation is 14%. Which of the following is the most likely explanation for
this woman’s anemia?
a. Occult blood loss
b. Vitamin deficiency
c. Anemia of chronic disease
d. Sideroblastic anemia

Ans: c
Ref: Harrison 20th Edition, Page 687
Patients with chronic inflammatory or neoplastic disease often develop anemia of chronic disease. Cytokines
produced by inflammation cause a block in the normal recirculation of iron from reticuloendothelial cells (which
pick up the iron from senescent red blood cells) to the red cell precursors (normoblasts). The peptide hepcidin is
felt to be the main mediator of
the effect. This defect in iron reutilization causes a drop in the serum iron concentration and a normocytic or
mildly microcytic anemia. The inflammatory reaction, however, also decreases the iron-binding capacity (as
opposed to iron-deficiency anemia, where the iron-binding capacity is elevated), so the saturation is usually
between 10% and 20%. The anemia is rarely severe (Hgb rarely <8.5 g/dL). The hemoglobin and hematocrit will
improve if the underlying process is treated.
Diseases not associated with inflammation or neoplasia (ie, diabetes, hypertension, etc.) do not cause anemia of
chronic disease. Blood loss causes a lower serum iron level, an elevated iron-binding capacity, and a lower iron
saturation. The serum ferritin (low in iron deficiency, normal or high in anemia of chronic disease) will usually
clarify this situation. Vitamin B12 and folate deficiencies are associated with macrocytic anemia. Sideroblastic
anemia can be either microcytic or macrocytic
(Occasionally with a dimorphic population of cells, some small and some large), but is associated with an
elevated iron level. In addition, this patient’s history (which suggests an inflammatory polyarthritis) would not be
consistent with sideroblastic anemia. The diagnosis of sideroblastic anemia is made by demonstrating ringed
sideroblasts on bone marrow aspirate. In the anemia of chronic renal insufficiency, caused by erythropoietin
deficiency, the iron studies are normal and the
red cells are normocytic.

3. A 60-year-old man presents with dull aching pain in the right flank. Physical examination reveals a firm
mass that does not move with inspiration. Laboratory studies show normal BUN, creatinine, and
electrolytes. Urinalysis shows hematuria. Hemoglobin is elevated at 18 g/dL and serum calcium is 11
mg/dL. What is the most likely diagnosis?
a. Polycystic kidney disease
b. Pheochromocytoma
c. Adrenal carcinoma
d. Renal cell carcinoma

Ans :d
Ref: Harrison 20th Edition, Page 616
Renal cell carcinoma is twice as common in men as women and tends to occur in the 50- to 70-year age group.
Many patients present with hematuria or flank pain, but the classic triad of hematuria, flank pain, and a palpable
flank mass occurs in only 10% to 20% of patients. Paraneoplastic syndromes such as erythrocytosis,
hypercalcemia, hepatic dysfunction, and

Paradise Institute & Self Study Centre Page 2


PARADISE WEEKLY MODEL TEST – CEE MD/MS BASED QAE SHRAWAN 20

fever of unknown origin are common. Surgery is the only potentially curable therapy; the results of treatment
with chemotherapy or radiation therapy for nonresectable disease have been disappointing. Interferon-alpha and
interleukin-2 produce responses (but no cures) in 10% to 20% of patients. Newer tyrosine kinase inhibitors (eg,
sunitinib) are active against renal cell cancers and hold promise for more effective treatment. The prognosis for
metastatic renal cell carcinoma is dismal.
Pheochromocytoma can cause erythrocytosis and occasionally hypercalcemia but would not cause hematuria or
an intrarenal mass. Polycystic kidney disease can cause erythrocytosis because of erythropoietin production by
the cysts but would cause numerous bilateral cysts, not a solid mass.

4. Transudative and exudative pleural effusions are distinguished by measuring the lactate dehydrogenase
(LDH) and protein levels in the pleural fluid (Pleural fluid protein/serum protein >0.5, Pleural fluid
LDH/serum LDH >0.6, Pleural fluid LDH more than two-thirds the normal upper limit for serum). How
many above criteria are required to label it as exudative?
a. 1/3
b. 2/3
c. 3/3
d. None out of 3

Ans: a
Ref: Harrison 20th Edition, Page No: 2006
Transudative and exudative pleural effusions are distinguished by measuring the lactate dehydrogenase (LDH)
and protein levels in the pleural fluid. Exudative pleural effusions meet at least one of the following criteria,
whereas transudative pleural effusions meet none:
•Pleural fluid protein/serum protein >0.5
•Pleural fluid LDH/serum LDH >0.6
•Pleural fluid LDH more than two-thirds the normal upper limit for serum
5. Which of the following lymphoid malignancy is caused by Epstein Barr virus?
a. Gastric MALT lymphoma
b. Post–organ transplant lymphoma
c. Lymphoplasmacytic lymphoma
d. Diffuse large B-cell lymphoma

Ans: b
Ref: Harrison 19th Edition, Page No: 697

Paradise Institute & Self Study Centre Page 3


PARADISE WEEKLY MODEL TEST – CEE MD/MS BASED QAE SHRAWAN 20

6. A 45 years old male presented with jaundice and ascites. On evaluation, his serum bilirubin was 2.1
mg/dl, serum albumin was 3.4 g/dl, INR was 1.8, had controlled ascites and minimal HE. His Child Pugh
class would be
a. A
b. B
c. C
d. D

Ans: c
Ref: Harrison’s Principles of Internal Medicine; 20th Edition, Page No: 2337

7.The score system used to estimate the risk of ischemic stroke in the first two days after TIA is:
a. NIHSS
b. CHA2DS2-VASc
c. HAS-BLED
d. ABCD2

Paradise Institute & Self Study Centre Page 4


PARADISE WEEKLY MODEL TEST – CEE MD/MS BASED QAE SHRAWAN 20

Ans: d
Ref: Authors: Karen L Furie et al., Initial evaluation and management of TIA and minor ischemic stroke
ABCD2 score:
The ABCD2 score can be used to estimate the risk of ischemic stroke in the first two days after TIA. The score is
tallied as follows:
A. Age:
▪ ≥60 years=1 point
▪ <60 years=0 points
B. Blood pressure elevation when first assessed after TIA:
▪ Systolic ≥140 mmHg or diastolic ≥90 mmHg=1 point
▪ Systolic <140 mmHg and diastolic <90 mmHg=0 points
C. Clinical features:
▪ Unilateral weakness=2 point
▪ Isolated speech disturbance=1 point
▪ Other= 0 point
D.
I. Duration of TIA symptoms:
▪ ≥60 minutes=2 points
▪ 10 to 59 minutes=1 point
▪ <10 minutes=0 point
II. Diabetes:
▪ Present = 1 point
Absent = 0 point

8. Bone marrow density 2 SD below the reference level is labeled as:


a. Osteopenia
b. Normal level
c. Osteoporosis
d. Severe osteoporosis

Ans: a
Ref: Authors: Joel S Finkelstein et al., Clinical manifestations, diagnosis and evaluation of osteoporosis in
men
• Normal: A value for BMD within 1.0 SD of the young adult female reference mean (T-score greater than
or equal to -1.0 SD).
• Low bone mass (osteopenia): A value for BMD more than 1.0 but less than 2.5 SD below the young adult
female reference mean (T-score less than -1 and greater than -2.5 SD).
• Osteoporosis: A value for BMD 2.5 or more SD below the young adult female reference mean (T-score
less than or equal to -2.5 SD).
• Severe (established) osteoporosis: A value for BMD more than 2.5 SD below the young adult female
reference mean in the presence of one or more fragility fractures

9. Cardiomegaly and ophthalmoplegia are seen due to the deficiency of:


a. Riboflavin
b. Niacin
c. Thiamine
d. Pyridoxine

Paradise Institute & Self Study Centre Page 5


PARADISE WEEKLY MODEL TEST – CEE MD/MS BASED QAE SHRAWAN 20

Ans: c
Ref: Harrison’s Principles of Internal Medicine; 20th Edition, Page No: 2310

10.Which of the following proteins does not cause secretion of gastric acid?
a. Acetylcholine
b. Caffeine
c. Histamine
d. Somatostatin

Ans: d
Ref: Harrison’s Principles of Internal Medicine; 20th Edition, Chap. 287
Gastric parietal cells create hydrochloric acid through a process of oxidative phosphorylation involving the H+-
K+-ATPase pump. For each molecule of hydrochloric acid produced, a bicarbonate ion is released into the gastric
venous circulation, creating the “bicarbonate tide.” Control of gastric acid secretion is primarily under the control
of the parasympathetic system. Postganglionic vagal fibers stimulate muscarinic receptors on parietal cells to
increase acid secretion. In addition, cholinergic
stimulation increases gastrin release from antral G cells as well as increasing the sensitivity of parietal cells to
circulating gastrin. Gastrin is the most potent stimulus of gastric acid secretion and is released from antral G cells
in response to cholinergic stimuli. Histamine is also a potent stimulus for gastric acid secretion. It is stored in
enterochromaffin like cells in the oxyntic glands of the stomach. Stimuli for histamine release include gastrin and
acetylcholine. Finally, caffeine stimulates gastrin release and thus increases acid secretion.
The most important protein produced in the stomach for inhibition of acid secretion is somatostatin. It is produced
in the D cells of the antrum, and its release is stimulated by a fall in the gastric pH to less than 3.0. Further
inhibition of gastric acid secretion is mediated by intestinal peptides secreted from the duodenum in response to
acid pH. These peptides include gastric inhibitory peptide and vasoactive intestinal peptide. Finally,
hyperglycemia and hypertonic fluids in the duodenum also inhibit gastric acid secretion through mechanisms that
are unknown.

Paradise Institute & Self Study Centre Page 6


PARADISE WEEKLY MODEL TEST – CEE MD/MS BASED QAE SHRAWAN 20

11. Differentiating the ACTH dependent Cushing's syndrome, which of the following feature is more
prominently seen in ACTH secreting pituitary tumor as compared to ectopic ACTH secretion
a. Rapid onset
b. Pigmentation
c. Severe myopathy
d. None of the above

Ans: d
Ref: Harrison’s Principles of Internal Medicine; 20th Edition, Page No: 2681

12. A 35-year-old political man who has been imprisoned for 3 years develops slowly progressive back pain
eventually accompanied by fatigue, intermittent fevers, night sweats, weight loss, and pain in the right foot
that progresses to weakness of the right leg. His WBC count and platelet count are normal but he has a
normochromic, normocytic anemia. Renal and hepatic functions are normal. What is the most likely
diagnosis?
a. Multiple myeloma
b. Acute myeloid leukemia (AML)
c. Spinal tuberculosis (Pott disease)
d. Transverse myelitis

Ans: c
Ref: Harrison 20th Edition, Page No: 2139-40
• Back pain is one of the most common diagnoses one encounters in medical practice. When it is
accompanied by systemic symptoms of fever and night sweats and eventually neurologic deficits one must
consider spinal tuberculosis (Pott disease) or a bacterial diskitis. Any patient diagnosed with TB should
also be checked for HIV infection. This patient’s 3 years of livingin jail increased his risk of developing
TB.
• Multiple myeloma certainly can present with back pain and bone pain but it is usually a disease of older
people with the median age at onset of 66. It is often associated with renal dysfunction.
• AML does share the symptoms of fatigue and weight loss although joint and bone pain is less common.
Most patients with AML present with symptoms related to their pancytopenia and this patient was

Paradise Institute & Self Study Centre Page 7


PARADISE WEEKLY MODEL TEST – CEE MD/MS BASED QAE SHRAWAN 20

reported to have a normal WBC count. This is also a disease of older patients generally with the median
age of onset of 65.
• Transverse myelitis typically has a clearly defined sensory level, bilateral symptoms and autonomic
symptoms including bladder dysfunction, symptoms which are not mentioned with regard to the patient in
question.

13. A 32-year-old woman has a long history of recurrent aphthous oral ulcers. In the past 2 months she has
had recurrent genital ulcers. She now presents with a red painful eye that was diagnosed as anterior
uveitis. What is the most likely diagnosis?
a. Herpes simplex
b. HIV infection
c. Behçet disease
d. Diabetes mellitus

Ans: c
Ref: Harrison 20th Edition, Page No: 3133
This patient has classic Behçet disease, which occurs more commonly in Asians. Behçet disease is a multisystem
disorder that usually presents with recurrent oral and genital ulcers. One-fourth of patients develop superficial or
deep vein thrombophlebitis. Iritis, uveitis, and nondeforming arthritis are common. Blindness, aseptic meningitis,
and CNS vasculitis may occur. Rare complications include pulmonary artery aneurysms and GI inflammation
which may lead to
perforation. Mucocutaneous lesions are usually treated with topical corticosteroids. Immunosuppressive therapy
is recommended for patients with threatened blindness or CNS disease. The oral lesions of herpes simplex
infection occur over the lips; anterior uveitis would be very uncommon. The mucocutaneous lesions of HIV
infection are usually caused by Candida and are easily distinguishable from aphthous ulcers. Neither diabetes nor
lupus would cause genital ulcers or anterior uveitis.

14. Shuffling gait is seen in:


a. Arthropathy
b. Parkinson’s disease
c. Sensory neuropathy
d. Cerebellar disease

Ans: a
Ref: Davidson’s Medicine 22nd Edition, page No: 172
• Antalgic: Arthropathy
• Waddling: Proximal myopathy
• Stamping: Sensory neuropathy
• Foot drop: Peripheral neuropathy or radiculopathy
• Ataxic: Sensory neuropathy or cerebellar disease
• Shuffling/festination: Parkinson’s disease
• Marche à petits pas: Small-vessel cerebrovascular disease
• Hemiplegic: Cerebral hemisphere lesion
• Apraxic: Bilateral hemisphere lesions
Marche à petits pas (“gait with little steps”) is a type of gait disorder characterized by an abnormal short stepped
gait with upright stance (in strict sense, as opposed to generally stooping short-stepped gait of Parkinson's
disease), seen in various neurological (or sometimes muscular) disorders. It can be further differentiated from

Paradise Institute & Self Study Centre Page 8


PARADISE WEEKLY MODEL TEST – CEE MD/MS BASED QAE SHRAWAN 20

"Parkinsonian gait" by normal arm swing (as opposed to no arm swing in Parkinsonism). This is associated with
frontal lobe white matter lesions.

15. A 59-year-old man undergoes coronary bypass surgery. He receives vancomycin prophylactically for 24
hours. On the ninth postoperative day, he develops a fever of 39.8°C with a HR 115 beats/min and a BP of
105/65 mm Hg. The surgical site is healing well with no redness or discharge. His WBCs 14,000/mm3 and
urinalysis reveals many white blood cells per high-power field. Blood and urine cultures grow a non–
lactose fermenting oxidase-positive gram-negative rod. Which of the following antibiotics is most
appropriate to treat this infection?
a. Moxifloxacin
b. Imipenem
c. Trimethoprim-sulfamethoxazole
d. Tigecycline

Ans: b
Ref: Harrison 20th Edition, Page No: 499
• The patient has a health care–associated UTI complicated by gram-negative bacteremia. The complete
identification of gram-negative rods might take 48 hours. Knowing the ability of the growing bacteria to
ferment lactose might help in the early prediction of the likely pathogen at hand. Among lactose fermenting
gram-negative rods, Enterobacteriaceae such as E coliare most common. Among non–lactose-fermenting
oxidase-positive gram-negative bacteria,
• Pseudomonas aeruginosa is most common. Ceftriaxone, imipenem, and trimethoprim sulfamethoxazole can
be used to treat UTIs while moxifloxacin and tigecycline do not achieve highenough concentration in urine to
be used for this indication. Of the listed antibiotics, imipenem, whichis a carbapenem beta-lactam antibiotic,
is the only one with anti-pseudomonal activity. Antibioticswith anti-pseudomonal activity include certain
penicillins (piperacillin/tazobactam andticarcillin/clavula-nate), cephalosporins (ceftazidime and cefepime),
carbapenems (imipenem,meropenem, and doripenem), fluoroquinolones (ciprofloxacin and levofloxacin),
andaminoglycosides (gentamicin, tobramycin, and amikacin).

16. A 34-year-old man presents with substernal discomfort. The symptoms are worse after meals,
particularly a heavy evening meal, and are sometimes associated with hot/sour fluid in the back of the
throat and nocturnal awakening. The patient denies difficulty swallowing, pain on swallowing, or weight
loss. The symptoms have been present for 6 weeks; the patient has gained 9kgs in the past 2 years. Which
of the following is the most appropriate initial approach?
a. Therapeutic trial of ranitidine or omeprazole
b. Esophagogastroduodenoscopy
c. CT scan of the chest
d. Coronary angiography

Ans: a
Ref: Harrison 20th Edition, Page No: 1162
• In the absence of alarm symptoms (such as dysphagia, odynophagia, weight loss, or gastrointestinal
bleeding), a therapeutic trial of acid reduction therapy is reasonable. Mild to moderate gastrointestinal
reflux disease (GERD) symptoms often respond to H2 blockers.
• More severe disease, including erosive esophagitis, usually requires proton-pump inhibitor therapy for 8
weeks to ensure healing. If the patient has recurrent symptoms or symptomatic GERD for over 5 years,
endoscopy is indicated to rule out Barrett esophagus (intestinal metaplasia of the lower esophagus).
• Barrett esophagus is a premalignant condition, and most patients receive surveillance EGD every 2 to 3
years, although evidence of mortality benefit from this approach is not available. In the absence of alarm

Paradise Institute & Self Study Centre Page 9


PARADISE WEEKLY MODEL TEST – CEE MD/MS BASED QAE SHRAWAN 20

symptoms, a therapeutic trial is generally favored over more expensive diagnostic studies (endoscopy, CT
scan). Classic symptoms of GERD do not mandate an evaluation for coronary artery disease unless other
features suggest this diagnosis.

17. New York Heart Association (NYHA) III means:


a. Slight limitation during ordinary activity
b. Unable to undertake physical activity without symptoms; symptoms may be present at rest
c. No limitation during ordinary activity
d. Marked limitation of normal activities without symptoms at rest

Ans: d
Ref: Davidson’s Medicine 22ndd Edition, page No: 539
New York Heart Association (NYHA) functional classification
▪ Class I No limitation during ordinary activity
▪ Class II Slight limitation during ordinary activity
▪ Class III Marked limitation of normal activities without symptoms at rest
▪ Class IV Unable to undertake physical activity without symptoms; symptoms may be present at rest

18. A 60-year-old man had an anterior myocardial infarction 3 months ago. He currently is asymptomatic
and has normal vital signs and a normal physical examination. His echocardiogram shows a mildly
depressed ejection fraction of 40%. He is on an antiplatelet agent, statin, and an ACE inhibitor. What
other category of medication would typically be prescribed for secondary prevention of myocardial
infarction?
a. Alpha-blocker
b. Beta-blocker
c. Calcium-channel blocker
d. Nitrates

Ans: b
Ref: Harrison 20th Edition, Page No: 2159
In patients who have had a myocardial infarction, beta-blockers are documented to lower the risk of myocardial
reinfarction and sudden death, whereas some calcium channel blockers may increase the risk. Alpha-blockers
have been associated with an increased risk of congestive heart failure. ACE inhibitors are beneficial in this
setting and should be continued.
Despite their decades-long use for the symptomatic treatment of angina, nitrates are not indicated for secondary
prevention of infarction. Recently, long-term use of some nonsteroidal anti-inflammatory drugs (including
naproxen sodium) has been associated with an increased risk of myocardial infarction.

Surgery

19. Harmonic scalpel uses which of the following to cut tissues?


a. Cold
b. Heat
c. Ultrasound
d. Fire

Ans: c
Ref: Bailey and Love’s Short Practice of Surgery 27th Edition, Page No: 104

Paradise Institute & Self Study Centre Page 10


PARADISE WEEKLY MODEL TEST – CEE MD/MS BASED QAE SHRAWAN 20

• The harmonic scalpel is an instrument that uses ultrasound technology to cut tissues while simultaneously
sealing them.
• It utilises a hand-held ultrasound transducer and scalpel which is controlled by a hand switch or foot
pedal.
• During use, the scalpel vibrates in the 20 000–50 000 Hz range and cuts through tissues, effecting
haemostasis by sealing vessels and tissues by means of protein denaturation caused by vibration rather
than heat (in a similar manner to whisking an egg white).

20. Which of the following is a major consequence of undescended testis?


a. Infection
b. Infertility
c. Malignancy
d. Both B and C

Ans: d
Ref: Bailey and Love’s Short Practice of Surgery 27th Edition, Page No: 125
The undescended testis
• A retractile testis reaches the base of the scrotum but retracts
• An undescended testis may be palpable or impalpable
• An ectopic testis lies outside the normal line of descent
• Palpable undescended testes undergo a single stage orchidopexy
• Impalpable undescended testes undergo a single stage orchidopexy
• Orchidopexy before 1 year of age improves fertility and may reduce the risk of malignancy
A palpable undescended testis ideally requires a day-case orchidopexy between 6 and 12 months of age. The
testis is mobilised through an inguinal incision, preserving the vas deferens and testicular vessels. The associated
patent processus vaginalis is ligated and divided at the internal ring and the testis is placed in a subdartos scrotal
pouch.

21. A 5 weeks old male child presents with vomiting. Olive felt at right upper quadrant of abdomen
suggests a diagnosis of:
a. Intussusception
b. Pyloric stenosis
c. Tracheo-esophageal fistula
d. Ileal atresia

Ans: b
Ref: Bailey and Love’s Short Practice of Surgery 27th Edition, Page No: 128
Infantile hypertrophic pyloric stenosis
• Most commonly affects boys aged 2–8 weeks
• Projectile vomiting after feeds
• Test feed or ultrasound to confirm the diagnosis
• Gastric peristalsis can be seen and an ‘olive’ felt
• Hypochloraemic metabolic alkalosis must be corrected before surgery
• Pyloromyotomy splits the hypertrophied muscle leaving the mucosa intact
IHPS can be diagnosed clinically. During a test feed there is visible gastric peristalsis passing from left to right
across the upper abdomen and in a relaxed baby the pyloric ‘tumour’ is palpable as an ‘olive’ in the right upper
quadrant. The diagnosis can be confirmed by an ultrasound, which shows the thickened pyloric muscle.

Paradise Institute & Self Study Centre Page 11


PARADISE WEEKLY MODEL TEST – CEE MD/MS BASED QAE SHRAWAN 20

22. A 74-year-old female is seen in the ED with altered mental status after a fall. She has a history of atrial
fibrillation and takes warfarin daily. On exam, she does not open her eyes, withdraws from pain, and make
incomprehensible sounds. What is her Glasgow Coma Scale (GCS) score?
a. 3
b. 4
c. 6
d. 7

Ans: d
Ref: Bailey and Love’s Short Practice of Surgery 27th Edition,
The patient has a Glasgow Coma Scale (GCS) score of 7 prior to intubation (eye opening = 1, verbal = 2, motor =
4). The GCS was developed to enable an initial assessment of the severity of head trauma. It is now also used to
standardize serial neurologic examinations in the early post-injury period.

23. Chloride content of Normal saline is (in mmol/L):


a. 111
b. 145
c. 154
d. 132

Answer: c
Ref: Bailey and Love’s Short Practice of Surgery 27th Edition, Page No: 281
Composition of crystalloid and colloid solutions (mmol/L):

24. While on a duty in ER, you are presented with mass casualties. Among them you are evaluating a
24year old male presented following Road traffic accident. On evaluation, his GCS is E3V3M5 and severe
facial trauma with mandibular fractures and blunt injuries to chest and abdomen. O/E HR 140/m, BP

Paradise Institute & Self Study Centre Page 12


PARADISE WEEKLY MODEL TEST – CEE MD/MS BASED QAE SHRAWAN 20

80/52mmHg, Spo2 88 (room air). Abdomen is tender and ecchymosis patches seen over the umbilical
region. In triage, color coding for this case is:
a. Red
b. Yellow
c. Green
d. Black

Ans: a
Ref: Bailey and Love’s Short Practice of Surgery 27th Edition, Page No: 412

25. Tenosynovitis of the abductor pollicis longus (APL) and extensor pollicis brevis (EPB) is known as:
a. Baby’s wrist
b. Soldier’s wrist
c. New mother’s wrist
d. Student’s wrist

Ans: c
Ref: Bailey and Love’s Short Practice of Surgery 27th Edition, Page No: 507, 508
De Quervain’s disease
• De Quervain’s disease is caused by tenosynovitis of the abductor pollicis longus (APL) and extensor
pollicis brevis (EPB) in the first dorsal wrist extensor compartment (1st EC).
• It is predominantly seen in middle-aged females and is associated with pregnancy (new mother’s wrist)
and inflammatory arthritis.
• The clinical features are radial wrist pain, tenderness, swelling and a positive Finkelstein’s test (pain over
the 1st EC associated with ulnar deviation of the wrist when the thumb is clasped in the palm).
• The management options are non-steroidal anti-inflammatories, splintage, steroid injections and surgical
release of the extensor retinaculum of the first dorsal compartment.
If surgery is considered, careful attention should be paid to fully releasing the APL and EPB, which frequently
consist of bundles of separate tendon slips that lie in separate sheaths

26. A 60-year-old otherwise healthy woman presents to her physician with a 3-week history of severe
headaches. A contrast CT scan reveals a small, circular, hypodense lesion with ringlike contrast
enhancement. Which of the following is the most likely diagnosis?
a. Brain abscess
b. High-grade astrocytoma
c. Metastatic lesion
d. Toxoplasmosis

Ans: c

Paradise Institute & Self Study Centre Page 13


PARADISE WEEKLY MODEL TEST – CEE MD/MS BASED QAE SHRAWAN 20

Ref: Bailey and Love’s Short Practice of Surgery 27th Edition,


The CT findings are consistent with any of the suggested lesions. However, the most likely diagnosis in an
immunocompetent patient is metastatic disease, which has an incidence of approximately 150,000 to 250,000
cases per year as compared to primary intracranial tumors, which have an incidence of 35,000 per year.
Roughly 15% to 30% of cancer patients develop intracranial metastases during the course of their disease. The
cancers that most frequently metastasize to the brain parenchyma include those of the lung, breast, kidney,
gastrointestinal (GI) tract, and melanomas.
Leukemia shows a predilection for the leptomeninges. A large majority of these lesions become symptomatic
owing to mass effect from white matter edema. Treatment is dependent on the number and size of the lesions and
the physical condition of the patient, but may include a combination of surgery, radiosurgery, and whole-brain
radiation therapy. Immunocompromised patients are at increased risk for toxoplasmosis and central nervous
system lymphomas.
Both immunocompetent and immunocompromised hosts can develop pyogenic brain abscesses, which typically
occur in the setting of known infection (which can spread either locally or hematogenously).

27. An 18-year-old man is admitted to the ED following a RTA. He is alert and fully oriented, but witnesses
to the accident report an interval of unresponsiveness following the injury. Skull films disclose a fracture of
the left temporal bone. Following x-ray, the patient suddenly loses consciousness and dilation of the left
pupil is noted. Which of the following is the most likely etiology of his diagnosis?
a. A ruptured aneurysm
b. A tear in his bridging veins
c. A tear in the middle meningeal artery
d. A ruptured arteriovenous malformation

Ans: c
Ref: Bailey and Love’s Short Practice of Surgery 26th Edition
Epidural hematomas are typically caused by a tear of the middle meningeal artery, and they may be associated
with linear skull fractures, usually in the temporal region. The lesion appears as a hyperdense biconvex mass
between the skull and the brain on CT scan. Clinical presentation is highly variable, and outcome depends largely
on promptness of diagnosis and surgical evacuation. The typical history is one of head trauma followed by a
momentary alteration in consciousness and
then a lucid interval lasting for up to a few hours. This is followed by a loss of consciousness, dilation of the pupil
on the side of the epidural hematoma, and then hemiparesis of the contralateral side. If treated promptly, outcome
is favorable in 85% to 90% of cases. Treatment consists of temporal craniectomy, evaluation of the hemorrhage,
and control of the bleeding vessel.

28. What is the emergent management of tension pneumothorax?


a. Tube thoracostomy in 5th intercostal space
b. Chest X-ray
c. Insert needle in 2nd intercostal space
d. Emergency room thoracotomy in unstable patient

Ans: c
Ref: Bailey and Love’s Short Practice of Surgery 27th Edition, Page No: 366
▪ Tension pneumothorax develops when a ‘one-way valve’ air leak occurs either from the lung or through
the chest wall.
▪ Air is sucked into the thoracic cavity without any means of escape, completely collapsing then
compressing the affected lung.

Paradise Institute & Self Study Centre Page 14


PARADISE WEEKLY MODEL TEST – CEE MD/MS BASED QAE SHRAWAN 20

▪ The mediastinum is displaced to the opposite side, decreasing venous return and compressing the opposite
lung.
▪ The most common causes are penetrating chest trauma, blunt chest trauma with parenchymal lung injury
and air leak that did not spontaneously close, iatrogenic lung punctures (e.g. due to subclavian central
venepuncture) and mechanical positive pressure ventilation.
▪ The clinical presentation is dramatic. The patient is increasingly panicky with tachypnoea, dyspnoea and
distended neck veins (similar to pericardial tamponade).
▪ Clinical examination may reveal tracheal deviation. This is a late finding and is not necessary to clinically
confirm diagnosis. There will also be hyper-resonance and absent breath sounds over the affected
hemithorax.
▪ Tension pneumothorax is a clinical diagnosis and treatment should never be delayed by waiting for
radiological confirmation.
Treatment consists of immediate decompression, initially by rapid insertion of a large-bore needle into the second
intercostal space in the midclavicular line of the affected hemithorax, then followed by insertion of a chest tube
through the fifth intercostal space in the anterior axillary line

29. Which of the following is recommended in management of the splenectomised patient?


a. Pneumococcal re-immunisation should be given annually
b. Life-long prophylactic penicillin V 500 mg weekly
c. Risk of cerebral malaria is drastically decreased
d. In septicaemia, patients should be resuscitated and given IV antibiotics to cover pneumococcus,
Haemophilus and meningococcus, according to local resistance patterns

Ans: d
Ref: Davidson’s Medicine 22ndd Edition, page No: 1028
Management of the splenectomised patient
▪ Vaccinate with pneumococcal, Haemophilus influenzae type B, meningococcal group C and influenza
vaccines at least 2–3 wks before elective splenectomy. Vaccination should be given after emergency
surgery but may be less effective
▪ Pneumococcal re-immunisation should be given at least 5-yearly and influenza annually. Vaccination
status must be documented
▪ Life-long prophylactic penicillin V 500 mg twice daily is recommended. In penicillin-allergic patients,
consider a macrolide
▪ Patients should be educated regarding the risks of infection and methods of prophylaxis
▪ A card or bracelet should be carried to alert health professionals to the risk of overwhelming sepsis
▪ In septicaemia, patients should be resuscitated and given IV antibiotics to cover pneumococcus,
Haemophilus and meningococcus, according to local resistance patterns
▪ The risk of cerebral malaria is increased in the event of infection
▪ Animal bites should be promptly treated with local disinfection and antibiotics, to prevent serious soft
tissue infection and septicaemia

30. Which of the following is a not benefit of laparoscopic surgery?


a. Less postoperative pain
b. Better cosmesis
c. Late return of normal physiological function
d. Shorter hospital stay

Ans: c
Ref: Bailey and Love 27th Edition, Page No: 87

Paradise Institute & Self Study Centre Page 15


PARADISE WEEKLY MODEL TEST – CEE MD/MS BASED QAE SHRAWAN 20

The benefits of laparoscopic surgery


• Less postoperative pain
• Better cosmesis
• Earlier return of normal physiological function
• Shorter hospital stay
• Earlier resumption of normal activities
The principles of safe laparoscopic surgery
• The umbilicus is preferred for primary trocar insertion
• An open or semi-open technique is preferred by most surgeons
• Open insertion away from the umbilicus is safer if there is a midline scar
• Secondary trocars should be inserted and removed under direct vision
• All trocars should be placed perpendicular to the abdominal wall
• All trocar sites above 5 mm in length should undergo closure of the fascial layer

31. Red flags for possible spinal pathology is:


a. Lumbar pain
b. 20-40 years of age
c. Single level of root sign
d. Saddle anaesthesia

Ans: d
Ref: Davidson’s Medicine 22ndd Edition, page No: 1073
Red flags for possible spinal pathology
History
• Age: presentation < 20 yrs or > 55 yrs
• Character: constant, progressive pain unrelieved by rest
• Location: thoracic pain
• Past medical history: carcinoma, tuberculosis, HIV, systemic corticosteroid use, osteoporosis
• Constitutional: systemic upset, sweats, weight loss
• Major trauma
Examination
• Painful spinal deformity
• Severe/symmetrical spinal deformity
• Saddle anaesthesia
• Progressive neurological signs/muscle-wasting
• Multiple levels of root signs

32. Which of the following is useful marker of resuscitation and physiological state?
a. Venous lactate
b. SPO2
c. Serum creatinine
d. Blood pressure

Ans: a
Ref: Bailey and Love 27th Edition, Page No: 327
Venous lactate
Venous lactate is a useful marker of resuscitation and physiological state.

Paradise Institute & Self Study Centre Page 16


PARADISE WEEKLY MODEL TEST – CEE MD/MS BASED QAE SHRAWAN 20

A normal lactate (<2 mmol/L) is a sign that the patient is probably resuscitated and suitable for early total care.
An elevated lactate (>3 mmol/L) suggests the patient is under-resuscitated and should either have a period of
further resuscitation, or DCS if surgery is urgent.
If a patient’s lactate is 2–3 mmol/L then the trend (upwards or downwards) should be noted and the other
physiological markers considered, to determine whether the patient is suitable for definitive surgical procedures.

33. A 22-year-old college student notices a bulge in his right groin. It is accentuated with coughing and
straining, but is easily reducible. Which of the following hernias follows the path of the spermatic cord
within the cremaster muscle?
a. Femoral
b. Direct inguinal
c. Indirect inguinal
d. Spigelian

Ans: c
Ref: Bailey and Love 27th Edition,
An indirect inguinal hernia leaves the abdominal cavity by entering the dilated internal inguinal ring and passing
along the anteromedial aspect of the spermatic cord. The internal inguinal ring is an opening in the transversalis
fascia for the passage of the spermatic cord; an indirect inguinal hernia, therefore, lies within the fibers of the
cremaster muscle.
In contrast, a direct inguinal hernia passes through a weakness in the floor of the inguinal canal medial to the
inferior epigastric artery (through Hesselbach’s triangle, made up of the lateral border of the rectus muscle, the
inferior epigastric vessels, and the inguinal ligament).
A femoral hernia passes directly beneath the inguinal ligament at a point medial to the femoral vessels. Neither a
direct inguinal hernia nor a femoral hernia lies within the cremaster muscle fibers. Spigelian hernias, which are
rare, protrude through an anatomic defect that can occur along the lateral border of the rectus muscle at its
junction with the linea semilunaris. An obturator hernia protrudes through the obturator foramen.

34.Majority of the patients with cholangiocarcinoma receive:


a. Surgical care
b. Adjuvant chemotherapy
c. Radiation therapy
d. Palliative care

Ans: d
Ref: Bailey and Love 27th Edition, Page No: 1208
Bile duct cancer (cholangiocarcinoma)
• Rare, but incidence increasing
• Most patients present with abnormal liver function tests or frank jaundice
• Diagnosis by ultrasound, CT or MRCP scanning
• The majority of patients receive palliative care only
• Complete surgical excision possible in <10%
• Prognosis poor: 90% die within 1 year, from liver failure or biliary sepsis
• Adjuvant chemoradiation therapy has a limited role
Malignant tumours of the gallbladder and extrahepatic biliary tree are uncommon. Gallbladder cancer is the
predominant type, accounting for 60–70% of cases with the remainder distributed between the intra- and
extrahepatic biliary trees. Unfortunately, owing to advanced stage at presentation, surgical resection, which offers
the best survival, is only possible for a minority of patients. Thus, for most patients treatment is generally
palliative in nature and survival is limited.

Paradise Institute & Self Study Centre Page 17


PARADISE WEEKLY MODEL TEST – CEE MD/MS BASED QAE SHRAWAN 20

35. A 48-year-old woman develops pain in the right lower quadrant whileplaying tennis. The pain
progresses and the patient presents to the ED later that day with a low-grade fever, a WBC count of
13,000/mm3, and complaints of anorexia and nausea as well as persistent, sharppain of the right lower
quadrant. On examination, she is tender in the rightlower quadrant with guarding and a bulge in that
location. An ultrasound isordered and shows an apparent mass in the abdominal wall. Which of
thefollowing is the most likely diagnosis?
a. Acute appendicitis
b. Cecal carcinoma
c. Hematoma of the rectus sheath
d. Torsion of an ovarian cyst

Ans: c
Ref: Bailey and Love 27th Edition
Hematomas of the rectus sheath are more common in the elderly, and a history of trauma, sudden muscular
exertion or anticoagulation can usually be elicited. The pain is of sudden onset and is sharp in nature. The
hematoma typically presents as an abdominal mass that does not change with contraction of the rectus muscles.
The diagnosis can be established with an ultrasound or CT scan showing a mass within the rectus sheath.
Management is conservative unless symptoms are severe and
bleeding persists, in which case embolization of the bleeding vessel by interventional radiology may be
performed. In rare cases, surgical evacuation of the hematoma and ligation of bleeding vessels may be required.

36. Push, Pringle, plug and pack is surgical management of trauma to:
a. Liver
b. Spleen
c. Pancreas
d. Bowel

Ans: a
Ref: Bailey and Love 27th Edition, Page No: 374
Liver trauma
• Blunt trauma occurs as the result of direct compression
• Penetrating trauma of the upper abdomen or lower thorax can damage the liver
• CT scanning is the investigation of choice in the stable patient
• Surgical management consists of push, Pringle, plug and pack
• The hepatic artery can be tied off but not the portal vein (which should be stented)
• Closed suction drainage should always be used.

Pediatrics

37. Which of the following muscle is measured in anthropometry?


a. Biceps
b. Triceps
c. Deltoid
d. Gluteus maximus

Ans: b
Ref: Short Textbook of Pediatrics, Suraj Gupte, 11th Edition, Page No: 131
▪ Triceps skin fold thickness is measured by a standard caliper (Lange, Herpenden or Best).

Paradise Institute & Self Study Centre Page 18


PARADISE WEEKLY MODEL TEST – CEE MD/MS BASED QAE SHRAWAN 20

▪ Tanner chart gives the normal values at different ages.


▪ On an average it exceeds 10 mm in 1 to 6 years age group.
▪ A measurement between 6 to 10 mm points to mild-moderate malnutrition and under 6 mm to severe
malnutrition.

38. Which of the following is not a minimal diagnostic criterion for Kwashiorkor?
a. Growth retardation
b. Hair changes (Flag sign)
c. Muscle wasting
d. Psychomotor change

Ans: b
Ref: Short Textbook of Pediatrics, Suraj Gupte, 11th Edition, Page No: 141
Kwashiorkor
Essential Features (Minimal Diagnostic Criteria)
▪ Growth retardation as evidenced by low weight and low height,
▪ Muscle wasting with retention of some subcutaneous fat,
▪ Psychomotor change as evidenced by mental apathy in the form of silent listless inertness, lack of interest
in the surroundings,
▪ Hypoalbuminemic pitting edema, at least over the pretibial region. Serum albumin should be less than 2.5
g/dl and cardiac, hepatic, renal and angioneurotic causes of edema should be ruled out.

39. Trisomy 18 is:


a. Down syndrome
b. Patau syndrome
c. Edward syndrome
d. Turner syndrome

Ans: c
Ref: Short Textbook of Pediatrics, Suraj Gupte, 11th Edition, Page No: 646

Paradise Institute & Self Study Centre Page 19


PARADISE WEEKLY MODEL TEST – CEE MD/MS BASED QAE SHRAWAN 20

40. A one-week-old, full-term infant born by uncomplicated vaginal delivery is brought to a pediatrician by
his mother, who notes that her daughter's skin appears yellow. She reports that the child cries several
times per day and sleeps 7-8 hours at night, uninterrupted. She has been breastfeeding exclusively and feels
that he has been feeding well. Which of the following is the best treatment for this condition?
a. Phototherapy
b. Phenobarbital administration
c. Plasma exchange
d. Continue normal breast feeding

Ans: d
The infant in this vignette most likely has breast milk jaundice. No intervention is needed, and the mother does
not need to stop breast feeding.
Breast milk jaundice occurs because the infant liver is not mature enough to process lipids. It presents between
the 4th and 7th day of life. If breast feeding stops, bilirubin levels fall rapidly; however, if breast feeding
continues, bilirubin levels will fall slowly. Distinct from breast milk jaundice, breastfeeding jaundice is the most
common cause of neonatal jaundice in the first week of life and occurs in 10% of births. It is characterized by
insufficient feeding, which leads to fewer bowel movements and, therefore, decreased bilirubin clearance. The
infant in this vignette has been breastfeeding normally.

41. A 7-month-old boy presents to the emergency room with three episodes of vomiting and severe
abdominal pain that comes and goes for the past two hours. The patient's most recent vomit in the hospital
appears bilious, and the patient had one stool that appears bloody and full of mucous. The mother explains
that one stool at home appear to be "jelly-like." On physical exam, a palpable mass is felt in the right lower
quadrant of the abdomen. What is the next best diagnostic test for this patient?
a. Peripheral blood culture
b. Kidney, ureter, bladder radiograph
c. Abdominal ultrasound
d. Exploratory laparotomy

Ans: c
Ref: Short Textbook of Pediatrics, Suraj Gupte, 11th Edition, Page No: 698
▪ The patient has symptoms most consistent with intussusception. The next and most sensitive and specific
diagnostic test would be an abdominal ultrasound.
▪ Intussusception is a process where a segment of the intestine slides into the adjacent intestinal lumen causing
bowel obstruction. A combination of abdominal pain, "currant-jelly" stools (blood and mucous), vomiting,
and an abdominal mass suggest intussusception.
The best initial diagnostic test is an abdominal ultrasound; however, despite being less sensitive and specific,
abdominal radiographs may also reveal intussusception

42.Neuroblastoma arises from:


a. Sympathetic ganglia
b. Adrenal cortex
c. Upper pole of kidney
d. All of the above

Ans: a
Ref: Short Textbook of Pediatrics, Suraj Gupte, 11th Edition, Page No: 563
Neuroblastoma

Paradise Institute & Self Study Centre Page 20


PARADISE WEEKLY MODEL TEST – CEE MD/MS BASED QAE SHRAWAN 20

▪ It is a malignant tumor arising from sympathetic ganglia or adrenal medulla.


▪ The common locations of neuroblastoma are the abdomen and chest.
Early metastases constitute the hallmark of the disease.

43. A neonate is examined in the nursery and found to have no anal orifice; only a small perineal fistulous
opening is visualized. Which of the following is associated with this abnormality?
a. Congenital pulmonary airway malformation
b. Hydrocephalus
c. Duodenal atresia
d. Congenital heart disease

Ans: d
Ref: Short Textbook of Pediatrics, Suraj Gupte, 11th Edition, Page No: 343
The neonate has low imperforate anus, the treatment of which is a perineal operation only. Imperforate anus is a
type of anorectal malformation that affects males and females with approximately the same frequency, occurring
in 1 in 5000 live births. It is due to failure of descent of the urorectal septum. Anorectal malformations can be
associated with other anomalies including heart disease, esophageal atresia (EA), abnormalities of the
lumbosacral spine, double urinary collecting systems, hydronephrosis, and communication between the rectum
and the urinary tract, vagina, or perineum.
Patients may present with a number of anomalies collectively known as the VACTERL syndrome: Vertebral
anomalies, Anal atresia, Cardiac defect (eg, ventricular septal defect [VSD]), TracheoEsophageal fistula, Renal
anomalies, and Limb defects (eg, radial dysplasia). A variety of surgical procedures have been devised to treat the
problem, depending on the type of anomaly (whether the rectum ends above or below the level of the levator ani
complex). However, even when anatomic integrity is established, the prognosis for effective toilet training is
poor. In 50% of cases, continence is never achieved. Congenital pulmonary airway malformation, hydrocephalus,
duodenal atresia, and corneal opacities have no significant association with congenital anorectal anomalies.

44. Pneumatocele is chest X-ray is classically associated with infection of:


a. Staphylococcus aureus
b. Group A streptococci
c. Adenovirus
d. Corona virus

Ans: a
Ref: Short Textbook of Pediatrics, Suraj Gupte, 11th Edition, Page No: 329
Demonstration of pneumatoceles is regarded pathognomonic of staphylococcal pneumonia.

45. A male toddler presented with difficulty in walking. There was history of recurrent falls and following
that on examination warmth and swelling of knee joint found. Most probable diagnosis is:
a. Scurvy
b. Hemophilia
c. Septic arthritis
d. Rheumatic fever

Ans: b
Ref: OP Ghai Pediatrics 9th Edition, Page No: 351
Hemophilia:
▪ Commonest hereditary clotting deficiency
▪ X-linked recessive disorders

Paradise Institute & Self Study Centre Page 21


PARADISE WEEKLY MODEL TEST – CEE MD/MS BASED QAE SHRAWAN 20

▪ Hemophilia A due to factor VIII deficiency


▪ Hemophilia B due to insufficient factor IX
Clinical features of hemophilia A and B are indistinguishable
▪ In mild cases the factor level is enough to prevent minor spontaneous bleeds and the children only
manifest, if they have surgery or severe trauma.
▪ In severe cases where factor levels are less than 1 %, repeated, spontaneous, debilitating joint bleeds lead
to severe handicap and intracranial bleeds can be life-threatening.
Treatment:
▪ Replacement therapy with concentrates of factor VIII or IX
▪ Judicious physiotherapy to prevent chronic joint disease,
▪ Counseling for injury prevention and
▪ Monitoring for inhibitor development.

46. What is the expected growth in length expected in a 5-year-old child?


a. 0.5 cm/year
b. 1 cm/year
c. 2 cm/year
d. 3 cm/year

Ans: d
Ref: Nelson pediatrics, 20th Edition, Page No: 87

47. Where are Epstein pearls located?


a. Abdomen
b. Face
c. Palate
d. Fingers & toes

Ans: c
Ref: Nelson pediatrics, 20th Edition, Page No: 3116
• Milia are superficial epidermal inclusion cysts that contain laminated keratinized material. The lesion is a
firm cyst, 1-2 mm in diameter, and pearly, opalescent white.
• Milia may occur at any age but in neonates are most frequently scattered over the face and gingivae and
on the midline of the palate, where they are called Epstein pearls.
• Milia exfoliate spontaneously in most infants and may be ignored; those that appear in scars or sites of
trauma in older children may be gently unroofed and the contents extracted with a fine-gauge needle.

48. A baby with multiple deformities, cleft lip, low set malformed ears, microcephaly, small eyes, scalp
defect and polydactyly seen in which syndrome:
a. Trisomy 21

Paradise Institute & Self Study Centre Page 22


PARADISE WEEKLY MODEL TEST – CEE MD/MS BASED QAE SHRAWAN 20

b. Trisomy 13
c. Trisomy 18
d. Trisomy 22

Ans: b
Ref: Nelson Pediatrics; 20th Edition; Page 611
Table: Chromosomal Trisomies and their clinical findings

Obstetrics and Gynecology

49. After delivery of a term infant with Apgar scores of 2 at 1 minute and 7 at 5 minutes, you ask that
blood from the umbilical arteries be collected for pH. The umbilical arteries carry which of the following?
a. Oxygenated blood to the placenta
b. Oxygenated blood from the placenta
c. Deoxygenated blood to the placenta
d. Deoxygenated blood from the placenta

Ans: c
Ref: Cunningham, pp 89-90
Deoxygenated fetal blood is returned directly to the placenta through the umbilical branches of the two
hypogastric arteries. The umbilical arteries exit through the abdominal wall at the umbilicus and continue by way
of the umbilical cord to the placenta. Deoxygenated blood circulates through the placenta then returns,
oxygenated, to the fetus via the umbilical vein. The umbilical arteries atrophy and obliterate within 3 to 4 days
after birth; remnants are called umbilical ligaments.

50. If hypertension is persisted beyond 6 months of delivery, this is:


a. Residual hypertension
b. Recurrent hypertension
c. Chronic hypertension
d. Recalcitrant hypertension

Ans: a
Ref: Dutta Obstetrics 8th Edition, Page No: 263
Residual hypertension:
▪ It may persist even after 6 months following delivery in about 50% cases.
▪ It is more related to familial diathesis and underlying thrombophilias (protein C, protein S deficiency,
antiphospholipid syndrome).
▪ Microvascular dysfunction due to insulin resistance is also there.
Paradise Institute & Self Study Centre Page 23
PARADISE WEEKLY MODEL TEST – CEE MD/MS BASED QAE SHRAWAN 20

51. During a routine return OB visit, an 18-year-old G1P0 patient at 23 weeks gestational age undergoes a
urinalysis. The dipstick done by the nurse indicates the presence of trace glucosuria. All other parameters
of the urine test are normal. Which of the following is the most likely etiology of the increased sugar
detected in the urine?
a. The patient has diabetes.
b. The patient has a urine infection.
c. The patient’s urinalysis is consistent with normal pregnancy.
d. The patient has underlying renal disease.

Ans: c
Ref: (Cunningham, p 124.)
The finding of glucosuria is common during pregnancy and usually is not indicative of a pathologic condition.
During pregnancy, there is an increase in the glomerular filtration rate and a decrease in tubular reabsorption of
filtered glucose. In fact, one of six women will spill glucose into the urine during pregnancy. If the patient has
risk factors for diabetes, such as obesity, previous macrosomia baby, advanced maternal age, or family history of
diabetes, the physician may want to screen for diabetes with a glucose challenge test. If the patient has a urinary
tract infection, the dipstick will show an increase in WBCs, the presence of nitrites and blood. A contaminated
urine sample would not be a cause of isolated glucosuria.

52. The incidence of scar rupture in hysterotomy scar is:


a. 0.2-1.5%
b. 4-9%
c. 12-15 %
d. 25 %

Ans: b
Ref: Dutta Obstetrics 8th Edition, Page No: 381
There is increased risk of scar rupture. Whereas the lower segment scar usually ruptures during labor that of
classical or hysterotomy scar ruptures during late pregnancy and labor. The incidence of scar rupture is about
0.2–1.5% in the former and about 4–9% in the latter.
Risk Factors for Scar Rupture
▪ More the number (>2) of prior cesarean delivery
▪ Interpregnancy interval <24 months
▪ Induced labor
▪ Augmentation of labor [with high dose oxytocin (>20 mU/min)]
▪ Women having single layer uterine closure in prior cesarean delivery compared to double layer closure.

53. Least common type of female pelvis is:


a. Gynecoid
b. Anthropoid
c. Android
d. Platypelloid

Ans: d
Ref: Dutta Obstetrics 8th Edition, Page No: 402
Variations of female pelvis:
The size and shape of the female pelvis differ so widely due to morphological factors such as developmental,
sexual, racial and evolutionary that it is indeed difficult to define what the features of a normal pelvis are.

Paradise Institute & Self Study Centre Page 24


PARADISE WEEKLY MODEL TEST – CEE MD/MS BASED QAE SHRAWAN 20

However, on the basis of the shape of the inlet, the female pelvis is divided into four parent types
▪ Gynecoid (50%)
▪ Anthropoid (25%)
▪ Android (20%)
▪ Platypelloid (5%)

54. A patient at 17 weeks gestation is diagnosed as having an intrauterine fetal demise. She returns to your
office 5 weeks later and her vital signs are: blood pressure 110/72 mm Hg, pulse 93 beats per minute,
temperature 36.38°C, respiratory rate 16 breaths per minute. She has not had a miscarriage, although she
has had some occasional spotting. Her cervix is closed on examination. This patient is at increased risk for
which of the
following?
a. Septic abortion
b. Recurrent abortion
c. Consumptive coagulopathy
d. Ectopic pregnancies

Ans: c
Ref: (Cunningham, p 787.)
In women with intrauterine fetal demise, labor usually occurs within 2 weeks. If the fetus is retained longer than 1
month, 25% of women can develop coagulopathy which is manifested by decreased fibrinogen, elevated fibrin
degradation products and decreased platelets. Septic abortions were more frequently seen during the era of illegal
abortions, although occasionally sepsis can occur if there is incomplete evacuation of the products of conception
in either a therapeutic or spontaneous abortion. However, since her cervix is closed and no Tissue has passed,
septic abortion is unlikely. Intrauterine fetal demise has no impact on future infertility or association with ectopic
pregnancies.

55. First line of treatment of stress incontinence in younger women is:


a. Conservative
b. Estrogen cream
c. Kelly surgery
d. Imipramine

Ans: a
Ref: Shaw Gynecology 16th Edition, Page No: 229, 230
Conservative treatment should be the first line of treatment, especially in younger women. It is cheap, has fewer
complications and does not compromise future surgery if so required.

56. Following a vaginal delivery, a woman develops fever, lower abdominal pain, and uterine tenderness.
She is alert, and her blood pressure and urine output are good. She started on broad-spectrum antibiotics,

Paradise Institute & Self Study Centre Page 25


PARADISE WEEKLY MODEL TEST – CEE MD/MS BASED QAE SHRAWAN 20

but continues to spike fevers 4 days postpartum. Which of the following is most closely tied to a decision to
proceed with hysterectomy?
a. Desires sterilization
b. Imaging showing pelvic abscess
c. Imaging showing gas in the myometrium
d. Septic shock

Ans: c
Ref: Cunningham, pp 666-667
With appropriate antibiotic coverage, most patients with endometritis are afebrile in 5 to 7 days.
Hysterectomy is usually reserved for women who have necrosis of the uterine incision typically depicted by gas
within the myometrium on imaging studies. Percutaneous drainage should be attempted for pelvic abscesses.
Signs of sepsis would be concerning, but first line of treatment would be supportive care and antibiotics. A
women desire for sterilization alone should not be the deciding factor for hysterectomy in this patient.

57. Therapeutic Pough of Douglas aspiration is done in:


a. Pelvic abscess
b. Ectopic pregnancy in haematocele
c. To detect malignancy in ascites with ovarian cyst
d. All of the above

Ans: a
Ref: Shaw Gynecology 16th Edition, Page No: 90
Aspiration of pouch of Douglas is required in the diagnosis of the following:
• Pelvic abscess
• Ectopic pregnancy in haematocele
• To detect malignancy in ascites with ovarian cyst
The only therapeutic purpose is to drain the pus in pelvic abscess

58. Most common and important cause of maternal death is:


a. Hemorrhage
b. Infection
c. Hypertension
d. Unsafe abortion

Ans: a
Ref: Dutta Obstetrics 8th Edition, Page No: 685
Important Causes of Maternal Deaths and Main Interventions
▪ Hemorrhage: 20-25 %
▪ Infection: 15-20 %
▪ Hypertension: 12-15 %
▪ Unsafe abortion: 10-13 %
59. Regarding pill time for Mini pill:
a. It has to be taken at the same time of the day
b. It has to be taken within 4 hours frame everyday
c. It has to be taken once every alternate day
d. It has to be taken within 12 hours frame everyday

Ans: a

Paradise Institute & Self Study Centre Page 26


PARADISE WEEKLY MODEL TEST – CEE MD/MS BASED QAE SHRAWAN 20

Ref: Dutta Obstetrics 8th Edition, Page No: 627


Progestin only contraception (POP/Mini pill)
POP is devoid of any estrogen compound. It contains very low dose of a progestin in any one of the following
form — levonorgestrel 75 μg, norethisterone 350 μg, desogestrel 75 μg, lynestrenol 500 μg or norgestrel 30 μg. It
has to be taken daily from the 1st day of the cycle.
The first pill has to be taken on the 1st day of the cycle and then continuously. It has to be taken regularly and at
the same time of the day. There must be no break between the packs. Delay in intake for more than 3 hours, the
woman should have missed pill immediately and the next one as schedule. Extra precaution has to be taken for
next 2 days.

60.Irregular early menstrual cycles soon after menarche is:


a. Menarche
b. Puberty menorrhagia
c. Insufficiency menorrhagia
d. Breakthrough menorrhagia

Ans: b
Ref: Shaw Gynecology 16th Edition, Page No: 61
Puberty menorrhagia: Soon after the menarche, the early menstrual cycles tend to be irregular and often
prolonged leading to severe anaemia.

ENT

61. Craniofacial dysfunction is seen in Le Fort:


a. I
b. II
c. III
d. IV

Ans: c
Ref: PL Dhingra and Shruti Dhingra ENT and HNS 7th Edition, Page No: 207
Fractures of maxilla
They are classified into three types:
1. Le Fort I (transverse) fracture runs above and parallel to the palate. It crosses lower part of nasal septum,
maxillary antra and the pterygoid plates.
2. Le Fort II (pyramidal) fracture passes through the root of nose, lacrimal bone, floor of orbit, upper part of
maxillary sinus and pterygoid plates. This fracture has some features common with the zygomatic
fractures.
3. Le Fort III (craniofacial dysjunction). There is complete separation of facial bones from the cranial bones.
The fracture line passes through root of nose, ethmofrontal junction, superior orbital fissure, lateral wall of
orbit, frontozygomatic and temporozygomatic sutures and the upper part of pterygoid plates.

62. A 38-year-old singer who underwent total thyroidectomy for multinodular goiter 6 months ago presents
with a reduction in her pitch range. She can no longer reach high pitches. Which of the following nerves
was most likely injured during her operation?
a. Superior laryngeal nerve
b. Bilateral recurrent laryngeal nerves
c. Unilateral recurrent laryngeal nerve
d. Hypoglossal nerve

Paradise Institute & Self Study Centre Page 27


PARADISE WEEKLY MODEL TEST – CEE MD/MS BASED QAE SHRAWAN 20

Ans: a
Ref: PL Dhingra and Shruti Dhingra ENT and HNS 7th Edition
The superior laryngeal nerve innervates the cricothyroid muscles. Therefore, superior laryngeal nerve injury,
which is also common after thyroid surgery, is manifested by voice weakness and difficulties with pitch,
particularly high notes. The recurrent laryngeal nerve innervates all other intrinsic muscles of the larynx, namely
those that open and close the vocal cords. Thus, unilateral recurrent laryngeal nerve injury can result in
hoarseness, voice changes, weak cough, or difficulty swallowing. Bilateral recurrent laryngeal nerve injury is
more serious and can result in complete airway obstruction and may require tracheostomy. Injury to the
hypoglossal nerve causes deviation of the tongue toward the side of the injury. The marginal mandibular branch
of the facial nerve innervates the muscles to the lower lip

63. Paranasal sinus of posterior group is:


a. Maxillary sinus
b. Frontal sinus
c. Anterior ethmoidal sinus
d. Sphenoid sinus

Ans: d
Ref: PL Dhingra and Shruti Dhingra ENT and HNS 7th Edition, Page No: 209
Paranasal sinuses are air-containing cavities in certain bones of skull. They are four on each side. Clinically,
paranasal sinuses have been divided into two groups:
▪ Anterior group: This includes maxillary, frontal and anterior ethmoidal. They all open in the middle
meatus and their ostia lie anterior to basal lamella of middle turbinate.
▪ Posterior group: This includes posterior ethmoidal sinuses which open in the superior meatus and the
sphenoid sinus which opens in sphenoethmoidal recess

64.Hypermotility disorder of esophagus is:


a. Nut cracker oesophagus
b. Cardiac achalasia
c. Gastroesophageal reflux
d. Amyotrophic lateral sclerosis

Ans: a
Ref: PL Dhingra and Shruti Dhingra ENT and HNS 7th Edition, Page No: 389
▪ Hypermotility disorder, e.g. cricopharyngeal spasm, diffuse oesophageal spasm, nut cracker oesophagus.
Hypomotility disorders, e.g. cardiac achalasia, gastroesophageal reflux, scleroderma, amyotrophic lateral
sclerosis.

65. A 27-year-old man presents to the ED with dizziness. He states he has experienced a sustained sensation
of the room spinning that is low grade and constant since this morning. The patient occasionally feels
nauseous and has been taking diphenhydramine to sleep which helps with his symptoms. Physical exam is
notable for a healthy man. The patient is sat upright, his head is turned slightly to the right, and he is laid
back flat rapidly. This does not provoke any symptoms even when repeated on the left side. A nystagmus is
notable on cranial nerve exam as well as bilateral decreased hearing. The patient’s tandem gait is unstable;
however, his baseline gait appears unremarkable despite the patient stating he has a sustained sensation of
imbalance. Which of the following is the most likely diagnosis?
a. Benign paroxysmal positional vertigo
b. Labyrinthitis

Paradise Institute & Self Study Centre Page 28


PARADISE WEEKLY MODEL TEST – CEE MD/MS BASED QAE SHRAWAN 20

c. Meniere disease
d. Vertebrobasilar stroke

Ans: b
This patient is presenting with vertigo, nystagmus, an abnormal tandem gait, decreased hearing, and a preceding
viral illness/cold without any risk factors suggesting a diagnosis of labyrinthitis.
• Labyrinthitis is a pathology that occurs secondary to infection/inflammation of the cochlear (hearing) and
vestibular (balance) apparatus which causes the key presentation of vertigo and hearing loss in a patient
after a viral illness. The hearing loss is a differentiating factor between labyrinthitis and vestibular neuritis
which only affects the vestibular portion of the nerve. There is no definitive treatment for this condition as
it self-limiting (other than symptomatic control with anticholinergic medications such as diphenhydramine
and meclizine).
• Benign paroxysmal positional vertigo presents with vertigo that is positional and can be provoked by the
Dix-Hallpike maneuver and treated by the Epley maneuver. The distinguishing feature is that this
pathology is associated with position changes but is not constant.
• Meniere disease presents with sudden-onset vertigo and nausea and vomiting in a relapsing/remitting
fashion with severe tinnitus, unilateral hearing loss (though the disease can be bilateral) and a sensation of
fullness in the ear. In between episodes, patients often feel well with a perpetual sensation of decreased
hearing.
• Vertebrobasilar stroke or a posterior circulation stroke would present with sudden onset and continuous
neurological deficits including cranial nerve deficits, weakness, vertigo, dysarthria, diplopia, dysphagia,
and multiple risk factors including hypertension, diabetes, increased age, and smoking among many
others. Though vertigo in this condition is continuous, epidemiologically it is unlikely in a younger,
healthier patient without any other neurological findings.

66. What is false about Gradenigo syndrome?


a. VIth nerve palsy
b. Ear discharge
c. Hearing loss
d. Retro-orbital pain

Ans : c
Ref: Dhingra ENT, 7th edition, page 88
• Gradenigo syndrome is the classical presentation of petrositis, and consists of a triad of (i) external
rectus palsy (VIth nerve palsy), (ii) deep-seated ear or retro-orbital pain (Vth nerve involvement)
and (iii) persistent ear discharge. It is uncommon to see the full triad these days.
• Persistent ear discharge with or without deep-seated pain in spite of an adequate cortical or modified
radical mastoidectomy also points to petrositis.Fever, headache, vomiting and sometimes neck rigidity
may also be associated. Some patients may get facial paralysis and recurrent vertigo due to involvement of
facial and statoacoustic nerves.
• Diagnosis of petrous apicitis requires both CT scan and MRI. CT scan of temporal bone will show bony
details of the petrous apex and the air cells while MRI helps to differentiate diploic marrow-containing
apex from the fluid or pus.
• Treatment: Cortical, modified radical or radical mastoidectomy is often required if not already done.

67. Nerve of Wrisberg is a part of:


a. Facial nerve
b. Vagus nerve
c. Vestibulocochlear nerve
Paradise Institute & Self Study Centre Page 29
PARADISE WEEKLY MODEL TEST – CEE MD/MS BASED QAE SHRAWAN 20

d. Hypoglossal nerve

Ans: a
Ref: Dhingra ENT 6th Edition, Page No: 449
Nerve of Wrisberg (syn. nervus intermedius)
It is a part of facial nerve and carries sensory and parasympathetic fibres.
The intermediate nerve, nervus intermedius, nerve of Wrisberg or Glossopalatine nerve, is the part of the facial
nerve (cranial nerve VII) located between the motor component of the facial nerve and the vestibulocochlear
nerve (cranial nerve VIII). It contains the sensory and parasympathetic fibers of the facial nerve. Upon reaching
the facial canal, it joins with the motor root of the facial nerve at the geniculate ganglion. Alex Alfieri postulates,
that the intermediate nerve should be considered as a separate cranial nerve and not a part of the facial nerve.

68. A 65-year-old man is diagnosed with squamous cell carcinoma to the right internal jugular lymph node
chain. Workup including CT and MRI head and neck are negative for a primary malignancy. Which of the
following is the next best step in the workup of this patient?
a. Modified barium swallow study
b. Laryngoscopy, nasopharyngoscopy, and esophagoscopy
c. Selective sampling of cervical lymph nodes
d. Modified radical neck dissection

Ans: b
Ref: Dhingra ENT 6th Edition
The patient has cervical lymph node metastases with an unknown primary. The most likely site of the primary is
in the upper aerodigestive tract. Therefore, the next step after negative imaging is to perform pan or triple
endoscopy which includes laryngoscopy, nasopharyngoscopy, and esophagoscopy (upper endoscopy).
Directed tissue biopsies should be performed during exam under anesthesia and pan-endoscopy. However,
selective cervical lymph node sampling is not indicated. Furthermore, modified radical neck dissection should not
be performed without first performing pan-endoscopy to attempt to identify the primary malignancy.
Modified barium swallow has no role in identifying malignancy and is primarily used in evaluating swallowing.
Neck ultrasound is of little utility given the negative CT and MRI scans.

69. At what age, is the adenoid largest in size?


a. Birth
b. 5 months of age
c. 6 – 7 years of age
d. 13 – 15 years of age

Ans : c
Ref: Dhingra ENT, 7th edition, page 275 - 276
• The nasopharyngeal tonsil, commonly called “adenoids”, is situated at the junction of the roof and
posterior wall of the nasopharynx.
• Adenoid tissue is present at birth, shows physiological enlargement up to the age of 6 years, and then
tends to atrophy at puberty and almost completely disappears by the age of 20.

Paradise Institute & Self Study Centre Page 30


PARADISE WEEKLY MODEL TEST – CEE MD/MS BASED QAE SHRAWAN 20

70. Which of the following modality detects temporal bone fracture is:
a. MRI
b. HRCT
c. X-ray
d. PET scan

Ans: b
Ref: High resolution computed tomography of the temporal bone: preliminary experience; by Tan TY1,
Lim CC, Boey HK ; Ann Acad Med Singapore.1994 Nov;23(6):869-75.
High Resolution Computed Tomography (HRCT) can demonstrate the detailed anatomy and pathology of
the temporal bone and is fast becoming an important imaging modality in the diagnosis and preoperative
management of our local patients with temporal bone diseases.
High-resolution computed tomography (HRCT) is a type of computed tomography (CT) with specific techniques
to enhance image resolution. It is used in the diagnosis of various health problems, though most commonly for
lung disease, by assessing the lung parenchyma.

Orthopedics

71. The cardinal signs of osteoarthritis includes all except:


a. Narrowing of the ‘joint space’
b. Subchondral sclerosis
c. Marginal osteophytes
d. Periarticular osteopenia

Ans: d
Ref: Apley Orthopedics; 9th Edition, Page No: 5
The cardinal signs of osteoarthritis
▪ Narrowing of the ‘joint space’
▪ Subchondral sclerosis
▪ Marginal osteophytes
▪ Subchondral cysts
▪ Bone remodeling

72. Crutch palsy is an example of:


a. Axonotmesis
Paradise Institute & Self Study Centre Page 31
PARADISE WEEKLY MODEL TEST – CEE MD/MS BASED QAE SHRAWAN 20

b. Neurotmesis
c. Neurapraxia
d. Neurulation

Ans: c
Ref: Apley Orthopedics; 9th Edition, Page No: 270
Neurapraxia
▪ Seddon (1942) coined the term ‘neurapraxia’ to describe a reversible physiological nerve conduction
block in which there is loss of some types of sensation and muscle power followed by spontaneous
recovery after a few days or weeks.
It is due to mechanical pressure causing segmental demyelination and is seen typically in ‘crutch palsy’, pressure
paralysis in states of drunkenness (‘Saturday night palsy’) and the milder types of tourniquet palsy.

73. Nerve root supplying the action of toe extension is:


a. L2
b. L4
c. L5
d. S1

Ans: c
Ref: Apley Orthopedics; 9th Edition, Page No: 229

Paradise Institute & Self Study Centre Page 32


PARADISE WEEKLY MODEL TEST – CEE MD/MS BASED QAE SHRAWAN 20

74. In intervertebral disc:


a. Central part is annulus fibrosus
b. Central part is avascular
c. Peripheral part contains nucleus pulposus
d. Central part containsconcentricfibrous tissues

Ans: b
Ref: Apley Orthopedics; 9th Edition, Page No: 489
INTERVERTEBRAL DISC
The disc consists of a central avascular nucleus pulposus – a hydrophilic gel made of protein-polysaccharide,
collagen fibres, sparse chondroid cells and water (88 per cent), surrounded by concentric layers of fibrous tissue –
the annulus fibrosus.
If the physicochemical state of the nucleus pulposus is normal, the disc can withstand almost any load that the
muscles can support; if it is abnormal, even small increases in force can produce sufficient stress to rupture the
annulus.

75. Treatment of Gun-Stock’ deformity is:


a. Circular osteotomy
b. Wedge osteotomy
c. Linear osteotomy
d. Focused osteotomy

Ans: b
Ref: Apley Orthopedics; 9th Edition, Page No: 371
Cubitus Varus (‘Gun-Stock’ Deformity)
▪ The deformity is most obvious when the elbow is extended and the arms are elevated. The most common
cause is malunion of a supracondylar fracture.
▪ The deformity can be corrected by a wedge osteotomy of the lower humerus but this is best left until
skeletal maturity.

76. Traction used for supracondylar fracture of humerus is?


a. Dunlop’s traction
b. Smith’s traction
c. Both
d. None

Ans: c
Ref: Maheshwari Orthopedics 5th Edition, Page 27

Paradise Institute & Self Study Centre Page 33


PARADISE WEEKLY MODEL TEST – CEE MD/MS BASED QAE SHRAWAN 20

77. Lunate dislocation is associated with injury of which nerve?


a. Median nerve
b. Ulnar nerve
c. Radial nerve
d. Any of the above

Ans: A, Median nerve


Ref: Maheshwari Orthopedics 5th Edition, Page 122 / Lunate dislocation by M.D. Lucas Meira Sarlo and
Dr Andrew Dixon et al in Radiopedia.
Lunate dislocation can lead to median nerve injury.
Lunate dislocations
• Uncommon traumatic wrist injury that require prompt management and surgical repair. The lunate is
displaced and rotated volarly. The rest of the carpal bones are in a normal anatomic position in relation to
the radius.
• These should not be confused with perilunate dislocations where the radiolunate articulation is preserved
and the rest of the carpus is displaced dorsally.
• Lunate dislocations typically occur in young adults with high energy trauma resulting in loading of a
dorsiflexed wrist. Overall, carpal dislocations comprise less than 10% of all wrist injuries. Lunate
dislocations are far less common than the less severe perilunate dislocation.
• Patients present with wrist pain following a fall. Volar wrist swelling is usually prominent. The swelling
often causes a decrease in 2-point discrimination in the median nerve distribution due to acute carpal
tunnel syndrome. Patients often prefer to hold their fingers in partial flexion due to pain on extension

78. Volkmann’s ischemic contracture is associated with?


a. Intertrochanteric femoral fracture
b. Supracondylar fracture of the humerus
c. Posterior dislocation of the knee
d. Traumatic shoulder separation

Ans: b
Ref: Ebnezar Orthopedics 4th Edition, Page No: 32, 33

Paradise Institute & Self Study Centre Page 34


PARADISE WEEKLY MODEL TEST – CEE MD/MS BASED QAE SHRAWAN 20

Volkmann’s ischemia/compartmental syndromes


Mubarak defined compartmental syndrome as an elevation of interstitial pressure in a closed osseofascial
compartment that results in microvascular compromise and may cause irreversible damage to the contents of the
space.
Sites
• Anterior and deep posterior compartments of the legs.
• Volar compartment of the forearm.
• Buttocks, shoulder, hand, foot, arm and lumbar paraspinous muscles are relatively rare sites.
COMPARTMENTAL SYNDROME OF FOREARM
• This is one of the most dreaded complications in orthopedics and ranges from mild ischemia to severe
gangrene.
• Early recognition and prompt remedial measures is the key to successful countering of this problem. This
is an orthopedic emergency.
• It is an ischemic necrosis of structures contained within the volar compartment of the forearm.
• It is common in children less than 10 years of age.
• Supracondylar fracture is the most common cause in children.
• Crush injuries of the forearm are the most common causes in adults.
• Occasionally fracture of both bones of forearm dislocation of the elbow, vascular injuries and subfascial
hematomas may be the cause.
More recently intra-arterial injections in drug addicts who lie on their forearm for prolonged periods in narcotized
conditions are mooted to be a cause

79. After a rotational injury to the knee, a patient hears a pop and notices that the knee is swollen within
60-90 minutes. This is most likely due to injury to the?
a. Patella
b. Anterior cruciate ligament
c. Posterior cruciate ligament
d. Lateral collateral ligament

Ans: b
Ref: Ebnezar Orthopedics 4th Edition, Page No: 248, 249
ANTERIOR CRUCIATE LIGAMENT (ACL) TEAR
• Of all the knee ligament injuries, ACL tear is the most common.
• The most common mode of injury is external rotation with abduction of the flexed knee or hyperextension
of knee in internal rotation.
• This is a disabling injury and the knee may immediately collapse and is painful.
• Popping sensation felt or heard at the time of injury signifies ligamentous injury (ACL tear). The patient
also tells that the knee "gave away" or buckled at the time of injury.
• Swelling of the knee could be either due to hemarthroses or traumatic synovitis and the distended knee is
held in partial flexion by the hamstrings.
• Sixty-seven percent of ACL tear is sports related

80. A 12-year-old boy presents to the emergency department after falling from his bike. He is holding his
right arm tenderly and complains of pain in his right wrist. Upon physical examination he is found to have
tenderness on the dorsal aspect of his wrist in between the extensor pollicis longus and the extensor pollicis
brevis. Given this presentation, which of the following is the most likely bone to have been fractured?
a. Scaphoid
b. Lunate

Paradise Institute & Self Study Centre Page 35


PARADISE WEEKLY MODEL TEST – CEE MD/MS BASED QAE SHRAWAN 20

c. Pisiform
d. Capitate

Ans: a
This presentation is consistent with a scaphoid bone fracture given the mechanism provided of falling onto his
right hand combined with tenderness in the anatomic snuff box on physical examination.
• The scaphoid bone is the most radial of the first row of carpal bones. Along with the trapezium bone it
forms the floor of the anatomic snuff box, which is an area bounded medially by the extensor pollicis
longus and laterally by the extensor pollicis brevis and abductor pollicis longus. Given its exposed
position, the scaphoid bone is the most commonly fractured carpal bone with the classic mechanism of
injury being fall on an outstretched hand. It is important to note that a common complication of proximal
scaphoid fractures is avascular necrosis due to the retrograde perfusion of the bone.
• The lunate bone is located centrally in the first row and is more likely to dislocate as opposed to fracture.
• The pisiform bone is a small bone located medially and forms part of Guyon's canal through which the
ulnar nerve runs.
• The capitate and trapezoid bones are located in the second row of carpal bones and are not within the
anatomic snuffbox.

Ophthalmology

81. What is the rotational movement around the anteroposterior axis, in which superior pole of cornea of
both the eyes tilts towards the right known as?
a. Dextrocycloversion
b. Dextro elevation
c. Levo cycloversion
d. Supra version

Ans: A
Ref: Khurana Ophthalmology, 6th edition, page 339
• Binocular movements. These are of two types: versions and vergences.
• a. Versions, also known as conjugate movements, are synchronous (simultaneous) symmetric
movements of both eyes in the same direction. These include:
i. Dextroversion. It is the movement of botheyes to the right. It results due to simultaneous
contraction of right lateral rectus and left medial rectus.
ii. Levoversion. It refers to movement of both eyes to the left. It is produced by simultaneous
contraction of left lateral rectus and right medial rectus.
iii. Supraversion. It is upward movement of both eyes in primary position. It results due to
simultaneous contraction of bilateral superior recti and inferior obliques.
iv. Infraversion. It is downward movement of both eyes in primary position. It results due to
simultaneous contraction of bilateral inferior recti and superior obliques.
v. Dextroelevation. It is movement to the right and upside. It is caused by simultaneous action of
right superior rectus and left inferior oblique.
vi. Dextrodepression. It is movement to the right and down side. It is caused by simultaneous action
of right inferior rectus and left superior oblique.
vii. Levoelevation. It is movement to the left and upside. It is caused by simultaneous action of left
superior rectus and right inferior oblique.
viii. Levodepression. It is movement to the left and down side. It is caused by simultaneous action of
left inferior rectus and right superior oblique.

Paradise Institute & Self Study Centre Page 36


PARADISE WEEKLY MODEL TEST – CEE MD/MS BASED QAE SHRAWAN 20

ix. Dextrocycloversion. It is rotational movement around the anteroposterior axis, in which superior
pole of cornea of both the eyes tilts towards the right.
x. Levocycloversion. It is just the reverse of dextrocycloversion. In this superior pole of cornea of
both the eyes tilts towards the left.

• b. Vergences, also called disconjugate movements, are synchronous and symmetric movements of both
eyes in opposite directions e.g.:
o 1. Convergence. It is simultaneous inward movement of both eyes which results from contraction of
the medial recti.
o 2. Divergence. It is simultaneous outward movement of both eyes produced by contraction of the
lateral recti.

82. Which organism causes ulcusserpens of cornea?


a. Haemophilus
b. Moraxella
c. Pneumococcus
d. Pseudomonas

Ans: C
Ref: Khurana Ophthalmology, 6th edition, page 102
• It is customary to reserve the term ‘hypopyon corneal ulcer’ for the characteristic ulcer caused by
Pneumococcus and the term ‘corneal ulcer with hypopyon’ for the ulcers associated with hypopyon due
to other organisms such as Staphylococci, Streptococci, Gonococci, Moraxella and Pseudomonas
pyocyanea.
• The characteristic hypopyon corneal ulcer caused by Pneumococcus is called ulcus serpens.

83. Herbert's pits are seen in:


a. Spring catarrh
b. Trachoma
c. Phlyctenular conjunctivitis
d. Sarcoidosis

Ans: b
Ref: Kanski’s Clinical Ophthalmology: A systematic approach; 8th edition; Page No: 138, 139
WHO grading of trachoma
▪ TF = trachomatous inflammation (follicular): five or more follicles (>0.5 mm) on the superior tarsal plate
▪ TI = trachomatous inflammation (intense): diffuse involvement of the tarsal conjunctiva, obscuring 50%
or more of the normal deep tarsal vessels; papillae are present
▪ TS = trachomatous conjunctival scarring: easily visible fibrous white tarsal bands
▪ TT = trachomatous trichiasis: at least one lash touching the globe
▪ CO = corneal opacity sufficient to blur details of at least part of the pupillary margin
Cicatricial trachoma
▪ Linear or stellate conjunctival scars in mild cases, or broad confluent scars (Arlt line) in severe disease.
▪ Although the entire conjunctiva is involved, the effects are most prominent on the upper tarsal plate.
▪ Superior limbal follicles may resolve to leave a row of shallow depressions (Herbert pits).
▪ Trichiasis, distichiasis, corneal vascularization and cicatricial entropion.
▪ Severe corneal opacification.
▪ Dry eye caused by destruction of goblet cells and the ductules of the lacrimal gland.

Paradise Institute & Self Study Centre Page 37


PARADISE WEEKLY MODEL TEST – CEE MD/MS BASED QAE SHRAWAN 20

84. Schwalbe's line is:


a. The posterior limit of the Descemet's membrane
b. The posterior limit of the Bowman's membrane
c. The anterior limit of the Descemet's membrane
d. The anterior limit of the Bowman's membrane

Ans: c
Ref: Kanski’s Clinical Ophthalmology: A systematic approach; 8th edition; Page No: 313
Schwalbe line
▪ This is the most anterior structure, appearing whitish to variably pigmented.
▪ Anatomically it demarcates the peripheral termination of Descemet membrane and the anterior limit of
the trabeculum.
▪ It may be barely discernible in younger patients. In contrast, there may be pigment deposits on or anterior
to the Schwalbe line – a Sampaolesi line – especially in heavily pigmented angles (e.g. pseudoexfoliation
syndrome). It may have a double-line configuration, when the posterior component may be mistaken for
the pigmented meshwork.

85. Sauce and cheese retinopathy is seen in:


a. Toxoplasmosis
b. CMV retinitis
c. Tuberculosis
d. Sarcoidosis

Ans: b
Ref: Parson Ophthalmology 22ndEdition, Page No: 615, 616
▪ Muddy appearance of the iris is seen in anterior uveitis or iridocyclitis. This muddy appearance of the iris
is due to the presence of fibrin on the anterior surface of the iris, giving the blurred and indistinct
appearance to the iris.
▪ Headlight in fog appearance on fundoscopic examination is seen in congenital toxoplasmosis.
▪ Sauce and cheese appearance on fundoscopic examination is seen in CMV retinitis.
▪ Koeppe nodules (iris nodules at the pupillary border) are characteristic of iridocyclitis or anterior uveitis.
▪ Busacca nodules (iris nodules at the collarette) are also characteristic of iridocyclitis.
▪ Mutton fat keratic precipitates are seen in granulomatous uveitis. Granular keratic precipitates are seen in
non-granulomatous uveitis. Red keratic precipitates are seen in haemorrhagic uveitis.
▪ Mutton fat KPs are composed of epithelioid cells and macrophages.
Earliest sign of anterior uveitis is aqueous flare and pathognomonic sign is keratic precipitate
86. Microaneurysms are the earliest feature of diabetic retinopathy. In which layer of retina are they seen?
a. Outer plexiform layer
b. Inner nuclear layer
c. Layer of rods and cones
d. Retinal pigment epithelium

Ans: b
Ref: Kanski’s Clinical Ophthalmology: A systematic approach; 8th edition; Page No: 520, 521
Microaneurysms
▪ Localized outpouchings, mainly saccular, of the capillary wall that may form either by focal dilatation of
the capillary wall where pericytes are absent, or by fusion of two arms of a capillary loop.
▪ Most develop in the inner capillary plexus (inner nuclear layer), frequently adjacent to areas of capillary
non-perfusion.

Paradise Institute & Self Study Centre Page 38


PARADISE WEEKLY MODEL TEST – CEE MD/MS BASED QAE SHRAWAN 20

▪ Loss of pericytes may also lead to endothelial cell proliferation with the formation of ‘cellular’
microaneurysms.
▪ Microaneurysms may leak plasma constituents into the retina as a result of breakdown in the blood–retinal
barrier, or may thrombose. They tend to be the earliest sign of diabetic retinopathy.
Arterial system
The central retinal artery, an end artery, enters the optic nerve approximately 1 cm behind the globe. It is
composed of three anatomical layers:
▪ The intima, the innermost, is composed of a single layer of endothelium resting on a collagenous zone.
▪ The internal elastic lamina separates the intima from the media.
▪ The media consists mainly of smooth muscle.
▪ The adventitia is the outermost and is composed of loose connective tissue.
Retinal arterioles arise from the central retinal artery. Their walls contain smooth muscle, but in contrast to
arteries the internal elastic lamina is discontinuous.
Capillaries
▪ Retinal capillaries supply the inner two-thirds of the retina, with the outer third being supplied by the
choriocapillaris.
▪ The inner capillary network (plexus) is located in the ganglion cell layer, with an outer plexus in the
inner nuclear layer.
▪ Capillary-free zones are present around arterioles and at the fovea (foveal avascular zone – FAZ).
▪ Retinal capillaries are devoid of smooth muscle and elastic tissue.

87. Epiphora means:


a. Cerebrospinal fluid running from nose after fracture of anterior cranial fossa
b. A presenting feature of a cerebral tumour
c. An abnormal flow of tears due to obstruction of the lacrimal duct
d. Eversion of lower eyelid following injury

Ans: c
Ref: Kanski’s Clinical Ophthalmology: A systematic approach; 8th edition; Page No: 65
Epiphora is the overflow of tears at the eyelid margin; strictly, it is a sign rather than a symptom. There are two
mechanisms:
• Hypersecretion secondary to anterior segment disease such as dry eye (‘paradoxical watering’) or
inflammation. In these cases watering is associated with symptoms of the underlying cause, and treatment
is usually medical.
• Defective drainage due to a compromised lacrimal drainage system; this may be caused by:
o Malposition (e.g. ectropion) of the lacrimal puncta.
o Obstruction at any point along the drainage system, from the punctal region to the valve of Hasner.
o Lacrimal pump failure, which may occur secondarily to lower lid laxity or weakness of the
orbicularis muscle (e.g. facial nerve palsy).
Anesthesia

88. What is not an ABG finding which is an indication of mechanical ventilation?


a. p (A - a) O2 gradient >350 mm Hg on 100% oxygen
b. pCO2 >50 mm Hg
c. pH <7.25
d. pO2/FiO2 <400 mm Hg

Ans : D
Ref: Ajay Yadav Anesthesia, 6th edition, page 278

Paradise Institute & Self Study Centre Page 39


PARADISE WEEKLY MODEL TEST – CEE MD/MS BASED QAE SHRAWAN 20

• Putting a patient on ventilator is more or less a clinical criterion however the general guidelines are:
• On the basis of blood-gas analysis:
o pO2 <50 mm Hg on room air or <60 mmHg on FiO2 (inspired oxygen)> 0.5 (50%).
o pH <7.25 (acute respiratory failure).
o pCO2 >50 mm Hg.
o pOifFiO2 <250 mm Hg (normal >400)
o p (A - a) 0 2 gradient >350 mm Hg on 100%oxygen.
• On the basis of pulmonary functions:
o Respiratory rate >35/ minute.
o Vital capacity <l5 mL/ kg.
o Dead space volume (VD/VT) >0.6 (60%).
o Peak negative pressure < -20 cm H2O.
o Tidal volume <5 mL/kg.
• Other.
o Excessive fatigue of respiratory muscles.
o Loss of protective airway reflexes which makes patient vulnerable for aspiration.
o Inability to cough adequately.

89. What is false about Mallampati grading?


a. Done to assess mouth opening
b. Faucial pillars are seen in grade I
c. Only soft palate is seen in grade III
d. Oral intubation can be done with difficulty in grade II

Ans: D
Ref: Ajay Yadav, Short Textbook of Anesthesia, 6th edition, page 53

• Mallampati grading: It is done to assess mouth opening. Patient is asked to open the mouth as wide as
possible and protrude the tongue. Depending on the structures seen by examiner the classification is as
follows:
• Class I: Faucial pillars, soft palate and uvula seen.
• Class II: Base of uvula and soft palate is seen.
• Class III: Only soft palate is seen.
• Class IV: Only hard palate is visible (modified Sampson and Young classification).
• In Mallampati grade (class) I and II oral intubation can be done comfortably, in grade III with
difficulty while oral intubation is not possible in grade IV

90. What is the potency of fentanyl as compared to morphine?


a. 0.1 times
b. 10 times

Paradise Institute & Self Study Centre Page 40


PARADISE WEEKLY MODEL TEST – CEE MD/MS BASED QAE SHRAWAN 20

c. 100 times
d. 1000 times

Ans: C
Ref: Ajay Yadav Anesthesia, 6th edition, page 117
• Fentanyl is the most common used opioid in anesthesia. Fentanyl along with bupivacaine or
ropivacaine is used as continuous infusion for painless labor and postoperative analgesia.
• It is 100 times more potent than morphine.
• The advantages of fentanyl are:
o Due to high lipid solubility it has rapid onset (2 to 5 min.) and rapid recovery (1-2 hours).
o It can be given by IM, IV, transmucosal, (fentanyl lollipop), transdermal, (fentanyl patch), intrathecal
and epidural route. Fentanyl patch provides analgesia for 72 hours.
o It is most cardiac stabile opioid.

91. What is the inhalational agent of choice in patients with renal diseases?
a. Desflurane
b. Enflurane
c. Isoflurane
d. Sevoflurane

Ans: A
Ref: Ajay Yadav Anesthesia, 6th edition, page 203
• Desflurane is the inhalational agent of choice in patient with renal diseases because it does not produce
fluoride.
• In case of non-availability of desflurane, isoflurane and halothane can be used safely. Sevoflurane
should be not be used in renal patients as it produces high levels of fluoride approaching renal
threshold of 50 μmol/L

92. What is the usual cause of apnea after spinal anesthesia?


a. High spinal
b. Medullary ischemia
c. Systemic toxicity
d. Total spinal

Ans: B
Ref: Ajay Yadav Anesthesia, 6th edition, page 166
Apnea after spinal anesthesia is usually due to severe hypotension causing medullary ischemia.

93. What does absence of post-tetanic count with non-depolarizing muscle blockers indicate?
a. Lack of block
b. Minimal block
c. Intense block
d. Recovery from block

Ans: C
Intense block
Ref: Ajay Yadav Anesthesia, 6th edition, page 68
• Post-tetanic count (PTC): A stimulus is given just after the tetanic stimulation.
• Tetanic stimulation will increase acetylcholine levels at neuromuscular junction.
Paradise Institute & Self Study Centre Page 41
PARADISE WEEKLY MODEL TEST – CEE MD/MS BASED QAE SHRAWAN 20

• This increased acetylcholine will replace some molecules of non-depolarizers from its binding site on
receptors producing a response while on the other hand the depolarizers produces relaxation by making
the membrane refractory due to continuous stimulation, therefore will not respond to increased
acetylcholine.
• Absence of PTC with non-depolarizers indicates a very intense block.

94.Which of the following is added to ketamine as preservative?


a. Benzethonium chloride
b. Benzyl alcohol
c. Egg lecithin
d. Sodium hydroxide
Ans: A
Ref: Ajay Yadav Anesthesia, 6th edition, page 112
• Ketamine produces dissociative anesthesia. Primary site of action is thalamo-neocortical
projection. Ketamine inhibits cortex (unconsciousness) and thalamus (analgesia) and stimulates
limbic system (emergence reaction and hallucinations).
• It is phencyclidine derivative. Available as solution of 10 mg/ mL and 50 mg/mL. Contains
preservative benzethonium chloride.

Psychiatry

95. What is a deeply ingrained pattern of behavior that includes modes of perception, relatingto and
thinking about oneself and the surrounding environment known as?
a. Affect
b. Attitude
c. Mood
d. Personality

Ans: D
Ref: Ahuja Psychiatry, 7th edition, page 113
Personality is defined as a deeply ingrained pattern of behaviour that includes modes of perception, relating to
and thinking about oneself and the surrounding environment. Personality traits are normal, prominent aspects of
personality. Personality disorders result when these personality traits become abnormal, i.e. become inflexible
and maladaptive, and cause significant social or occupational impairment, or significant subjective distress

96. A 64-year-old man is admitted to the psychiatric unit after an unsuccessful suicide attempt. Following
admission, he attempts to cut his wrists three times in the next 24 hours and refuses to eat or drink
anything. He is scheduled to have electroconvulsive therapy (ECT) because he is so severely depressed
that an antidepressant is deemed too slow acting. Which of the following side effects should the patient
be informed is most common after ECT?
a. Headache
b. Palpitations
c. Deep venous thromboses
d. Interictal confusion

Ans: A
Ref: (Kaplan and Sadock, pp 1071-1072.)
The most common complaints after ECT include headaches, nausea, and muscle soreness. Memory impairment
(both retrograde and anterograde) does occur but less frequently, though 75% of patients say that memory

Paradise Institute & Self Study Centre Page 42


PARADISE WEEKLY MODEL TEST – CEE MD/MS BASED QAE SHRAWAN 20

impairment is the worst adverse effect. Interictal confusion is quite uncommon. Likewise, cardiovascular changes
do occur, but they are rare and happen mostly in the immediate postictal period or during the seizure itself.

97. What is a person trying to kill herself but never intending to die by the act known as?
a. Attempted suicide
b. Presumed suicide
c. Suicide
d. Suicidal gesture

Ans: D
Ref: Ahuja Psychiatry, 7th edition, page 222
• Suicide is the model of psychiatric emergencies and is also the commonest cause of death among the
psychiatric patients.
• Suicide is a type of deliberate self-harm (DSH) and is defined as a human act of self-intentioned and self-
inflicted cessation (death). It ends with a fatal outcome. DSH is an act of intentionally injuring oneself,
irrespective of the actual outcome.
• An attempted suicide is an unsuccessful suicidal act with a nonfatal outcome. It is believed that 2-10% of all
persons who attempt suicide, eventually complete suicide in the next 10 years.
• A suicidal gesture, on the other hand, is an attempted suicide where the person performing the action never
intends to die by the act. However, some of these persons may accidentally die during the act. Attempted
suicide is more com mon in women while completed suicide is 2-4 times commoner in men.

98. What is thoughts being imposed by some external force on person’s passive mind known as?
a. Thought broadcasting
b. Thought diffusion
c. Thought insertion
d. Thought removal

Ans: C
Ref: Ahuja Psychiatry, 7th edition, page 55
First Rank Symptoms (SFRS) of Schizophrenia:
• Audible thoughts: Voices speaking out thoughts aloud or ‘thought echo’.
• Voices heard arguing: Two or more hallucinatory voices discussing the subject in third person.
• Voices commenting on one’s action.
• Thought withdrawal: Thoughts cease and subject experiences them as removed by an external force.
• Thought insertion: Experience of thoughts imposed by some external force on person’s passive mind.
• Thought diffusion or broadcasting: Experience of thoughts escaping the confines of self and as being
experienced by others around.
• ‘Made’ feelings or affect.
• ‘Made’ impulses.
• ‘Made’ volition or acts: In ‘made’ affect, impulses and volitions, the person experiences feelings, impulses or
acts which are imposed by some external force. In ‘made’ volition, for example, one’s own acts are
experienced as being under the control of some external force.
• Somatic passivity: Bodily sensations, especially sensory symptoms, are experienced as imposed on body by
some external force.
• Delusional perception: Normal perception has a private and illogical meaning.

Paradise Institute & Self Study Centre Page 43


PARADISE WEEKLY MODEL TEST – CEE MD/MS BASED QAE SHRAWAN 20

99. A 35-year-old woman is seeing a psychiatrist for treatment of her major depressive disorder. After 4
weeks on fluoxetine at 40 mg/day, her psychiatrist decides to try augmentation. Which of the following
is the most appropriate medication?
a. Lithium
b. Sertraline
c. An MAO inhibitor
d. Haloperidol

Ans: A
Ref: (Kaplan and Sadock, p 984.)
Lithium has been proven effective when added to an antidepressant in the treatment of refractory depression.
More than one mechanism of action is probably involved, although lithium’s ability to increase the presynaptic
release of
serotonin is the best understood. Other augmentation strategies include the use of thyroid hormones, stimulants,
estrogens, and light therapy. The combination of two SSRIs (in this case, fluoxetine and sertraline) or of an
MAOI and an SSRI is not recommended because of the risk of precipitating a serotonin syndrome

100. An 8-year-old male presents to his pediatrician with dry, cracking skin on his hands. His mother states
that this problem has been getting progressively worse over the past couple of months. During this time
period, she has noticed that he also has become increasingly concerned with dirtiness. He tearfully
admits to washing his hands many times a day because "everything has germs." When asked what
happens if he doesn't wash them, he responds that he just feels very worried until he does. With which
other condition is this disorder associated?
a. Tourette's syndrome
b. Obessive-compulsive personality disorder
c. Delusional disorder
d. Rett's disorder

Ans: a
This boy with repetitive hand-washing secondary to anxiety over germs has obsessive-compulsive disorder
(OCD), which is associated with Tourette's syndrome.
• OCD, a common ego-dystonic anxiety disorder that arises in childhood, is characterized by recurrent
intrusive, distressing thoughts ("obsessive"). The anxiety caused by these thoughts is alleviated by
repetitive behaviors ("compulsive"), which vary from patient to patient, depending on the specific
thoughts. Two of the most common forms are fears about contamination leading to hand-washing and
persistent doubts leading to various forms of checking.
• One key difference between OCD and obsessive-compulsive personality disorder (OCPD) is that
individuals with OCD do not see their behaviors as consistent with their normal beliefs (ego-dystonic)
whereas individuals with OCPD do (ego-syntonic).
• In delusional disorder, individuals have fixed, non-bizarre beliefs that persist for more than one month. It
is not associated with OCD.
• Rett's disorder, a pervasive developmental disorder, does develop in childhood but only affects females
(incompatible with life for males). Classically, individuals display repetitive hand-wringing but not OCD.

101. While attending a party, a 35-year-old widow noticed a male who physically resembled her deceased
husband. She introduced herself and began a conversation with the male while making sure not to
make mention of the resemblance. After the conversation, she felt feelings of affection and warmth to
the male similar to how her husband made her feel. Which of the following best explains the widow's
feelings towards the male?

Paradise Institute & Self Study Centre Page 44


PARADISE WEEKLY MODEL TEST – CEE MD/MS BASED QAE SHRAWAN 20

a. Transference
b. Countertransference
c. Sublimation
d. Projection

Ans: a
Ref: Niraj Ahuja Psychiatry 7th Edition, Page No: 214
The female is displaying transference when she feels affection and warmth towards the male who resembles her
deceased husband.
• Transference is the unconscious shifting of emotions associated with one person to another. Transference
typically occurs in doctor-patient relationships where the patient projects feelings onto the physician. The
reverse can also occur when the doctor projects feelings onto the patient, which is called
countertransference.
• Countertransference is when the physician projects feelings towards another person onto the patient.
• Sublimation is a mature ego defense when one replaces an unacceptable action with a similar, acceptable
action.
• Projection is an immature ego defense when one attributes unacceptable internal feelings to an external
source.

Radiology

102. Which of the following is the commonest location of Ewings sarcoma?


a. Epiphysis
b. Metaphysis
c. Diaphysis
d. Epi-metaphysis

Ans: c
Ref: Primer of Diagnostic Imaging, 5th edition, Page No: 311
Location of bone lesions according to anatomy:
• Epiphysis: typical are cartilaginous and articular lesions such as chondroblastoma or eosinophilic
granuloma (EG)
• Metaphysis: lesions of different causes (neoplastic eg. Osteosarcoma, inflammatory, metabolic) have
apredilection for the metaphysis (rich blood supply); therefore, this location alone is of limited differential
diagnostic value.
• Epiphyseal/metaphyseal region: giant cell tumors
• Diaphysis: Ewing’s sarcoma

103. The best way to determine the rotation in the chest X-ray is to look for the:
a. Position of the medial ends of each clavicle relative to the spinous process
b. Compare the latency of the both lung-fields.
c. Appearance of the ribs
d. Appearance of the scapula

Ans: a
Learning Radiology, Recognizing the basics, 3rd Edition
• Significant rotation (the patient turns the body to one side or the other) may alter the expected contours of
the heart and great vessels, the hila, and hemidiaphragms.

Paradise Institute & Self Study Centre Page 45


PARADISE WEEKLY MODEL TEST – CEE MD/MS BASED QAE SHRAWAN 20

• The easiest way to assess whether the patient is rotated toward the left or right is by studying the position
of the medial ends of each clavicle relative to the spinous process of the thoracic vertebral body between
the clavicles. If the spinous process appears to lie equidistant from the medial end of each clavicle on the
frontal chest radiograph, there is no rotation
• If the spinous process appears closer to the medial end of the left clavicle, the patient is rotated toward his
or her own right side.
• If the spinous process appears closer to the medial end of the right clavicle, the patient is rotated toward
her or his own left side.

104. Which of the following is a sign of pneumoperitoneum?


a. Football sign
b. Copula sign
c. Ligamentum teres sign
d. All of the above

Ans d
Ref: Learning Radiology, Recognizing the basic pattern. 3rd edition, page 411
Signs seen in pneumoperitoneum:
• Football sign = large pneumoperitoneum outlining entire abdominal cavity
• Double wall sign = Rigler sign = air on both sides of bowel caused by intraluminal gas and free air outside
(usually requires >1 L of free intraperitoneal gas and intraperitoneal fluid)
• Inverted V sign = outline of both lateral umbilical ligaments(containing inferior epigastric vessels)
• Urachus sign = outline of middle umbilical ligament
• Single large area of hyperlucency over the liver
• Oblique linear area of hyperlucency outlining the posteroinferior
• Outline of falciform ligament = long vertical line to the right of midline extending from ligamentum teres
notch to umbilicus; most common structure outlined
• Ligamentum teres notch = inverted V-shaped area of hyperlucency along undersurface of liver
• Ligamentum teres sign = air outlining fissure of ligamentum teres hepatis along posterior free edge of
falciform ligament seen as vertically oriented, sharply defined slit like/oval area of hyperlucency between
10th and 12th ribs within 2.5 to4.0 cm of right vertebral border, 2 to 7 mm wide and 6 to20 mm long
• Saddlebag/mustache/cupola sign = gas trapped below central tendon of diaphragm.

105. What is the shape of subdural hematoma in cross sectional imaging?


a. Hour glass shaped
b. Biconvex
c. Crescentic shaped
d. Lens shaped

Ans: c
Epidural haematoma: Typically lentiform (lens-shaped, biconvex, lemon-shaped) and do not cross sutures
as the periosteum crosses through the suture continuous with the outer periosteal layer.
Subdural haematoma: Typically crescentic (crescent moon-shaped, concave, banana-shaped) and more
extensive than EDH, with the internal margin paralleling the cortical margin of the adjacent brain. As these occur
in the subdural space, they cross sutures.

of the following does not make the right cardiac border in the Chest X – ray?
106. Which
a. Right atrium
b. Right Ventricle

Paradise Institute & Self Study Centre Page 46


PARADISE WEEKLY MODEL TEST – CEE MD/MS BASED QAE SHRAWAN 20

c. SVC
d. IVC

Ans: b
Learning Radiology, Recognizing the basics. 3rd edition, page 25
Normal right cardiac border seen in the frontal projection
• Superior venacava
• Ascending aorta (when tortuous)
• Right atrium
• inferior venacava
Normal right cardiac border seen in the frontal projection
• Aortic Knuckle
• Pulmonary Bay
• Left Atrial appendage
• Left ventricle

107. FLAIR sequences are used in:


a. USG
b. CT
c. MRI
d. Barium study

Ans: c
Fluid attenuation inversion recovery (FLAIR) is a special inversion recovery sequence with a long inversion time.
This removes signal from the cerebrospinal fluid in the resulting images. Brain tissue on FLAIR images appears
similar to T2 weighted images with grey matter brighter than white matter but CSF is dark instead of bright.
TR TE
(msec) (msec)
T1-Weighted 500 14
(short TR and TE)
T2-Weighted 4000 90
(long TR and TE)
Flair 9000 114
(very long TR and TE

108. Doppler is used to study:


a. Flow
b. Concentration
c. Softness
d. Liquefaction

Ans: a
Ref: Yang P. Applications of colour Doppler ultrasound in the diagnosis of chest diseases. (1997)
Respirology (Carlton, Vic.). 2 (3): 231-8.
Color flow Doppler is used frequently in sonography to semiquantitate overall blood flow to a region of interest.
Depiction of the general velocity and direction of blood flow within the heart and blood vessels is of primary

Paradise Institute & Self Study Centre Page 47


PARADISE WEEKLY MODEL TEST – CEE MD/MS BASED QAE SHRAWAN 20

importance in echocardiography and vascular ultrasound respectively. It also allows the generation of unique
phenomena such as the fluid color sign or the twinkling artifact, and allows the targeting of spectral Doppler for a
quantitative assessment of blood flow

Dermatology

109. Which of the following is associated with typical apple jelly color on diascopy?
a. Lepromatous leprosy
b. Lupus vulgaris
c. Scrofuloderma
d. Tuberculoid leprosy

Ans: B
Ref: Fitzpatrick’s Dermatology, 9th edition, page 2864
• Lesions of lupus vulgaris (LV) are usually solitary, but two or more sites may be involved simultaneously. In
patients with active pulmonary tuberculosis, multiple foci may develop. In approximately 90% of patients, the
head and neck are involved. LV usually starts on the nose, cheek, earlobe, or scalp and slowly extends onto
adjacent regions. Other areas are rarely involved. The initial lesion is a brownish-red, soft or friable macule or
papule with a smooth or hyperkeratotic surface.
On diascopy, the infiltrate exhibits a typical apple jelly color. Progression is characterized by elevation, a deeper
brownish color, and formation of a plaque. Involution in one area with expansion in another often results in a
gyrate outline border. Ulceration may occur. Hypertrophic forms appear as a soft nodule or plaque with a
hyperkeratotic surface

110. A 42-year-old woman presents to the ED with a rash. On examination she has a widespread
erythematous eruption with hemorrhagic oral crusting and conjunctivitis. The next day on the ward
she becomes very unwell with sheets of skin loss and a positive Nikolsky’s sign. Which of her drugs is
most likely to be responsible:
a. Codeine phosphate
b. oral contraceptive pill
c. ciclosporin
d. allopurinol

Ans: d
Ref: Rook’s Dermatology, 9th edition, page 4.23 – 4.24
This patient has developed toxic epidermal necrolysis (TEN), a life-threatening condition usually attributed to
drug reactions. In TEN there is widespread keratinocyte death throughout the epidermis leading to large areas of
skin loss. There is often a preceding fever and significant mucous membrane and eye involvement. TEN has 30–
40% mortality, most often from sepsis, and those who survive often have long term ocular complications. It is
thought that Stevens-Johnson syndrome (SJS) and TEN are spectrums of the same disorder. There is however
disagreement over whether erythema multiforme is also part of this spectrum. SJS may be caused by drug
reactions, viral infections or malignancy but in up to 50% of cases no cause is found. The rash of SJS is similar to
erythema multiforme with pronounced mucous membrane involvement. The treatment of SJS and TEN is
supportive with some dermatologists advocating corticosteroids and intravenous immunoglobulins.
Common causes of toxic epidermal necrolysis:
• non-steroidal anti-inflammatory medications
• sulphonamides
• beta-lactam antibiotics
• anticonvulsants

Paradise Institute & Self Study Centre Page 48


PARADISE WEEKLY MODEL TEST – CEE MD/MS BASED QAE SHRAWAN 20

• allopurinol.

111. White dermographism is pronounced in which of the following conditions?


a. Atopic dermatitis
b. Drug induced wheals
c. Cholinergic urticaria
d. Urticarial vasculitis

Ans: A
Ref: Rook’s Dermatology, 9th edition, page 42.10
White dermographism (due to capillary vasoconstriction following light stroking of the skin) occurs normally but
is particularly pronounced in atopic eczema.

112. What disease gives row of tombstones appearance in histopathology?


a. Bullous pemphigoid
b. Epidermolysis bullosa simplex
c. Pemphigus foliaceous
d. Pemphigus vulgaris

Ans: D
Ref: Fitzpatrick’s Dermatology, 9th edition, page 921

• The characteristic histopathologic finding in PV is a suprabasal blister with acantholysis. Just above the basal
cell layer, epidermal cells lose their normal cell-to-cell contacts and form a blister. Often, a few rounded up
(acantholytic) keratinocytes are in the blister cavity.
• The basal cells stay attached to the basement membrane, but may lose the contact with their neighbors; as a
result, they may appear to be a “row of tombstones,” symbolic of the potentially fatal prognosis of this
disease. Usually, the upper epidermis (from 1 or 2 cell layers above the basal cells) remains intact, as these
cells maintain their cell adhesion.

113. A one-day-old baby is referred with an extensive flat, vascular patch covering most of the left side of
the face. It extends from the mandible to the forehead, laterally to the ear and medially to the nasal
bridge. What investigation should be organized as a matter of urgency:
a. CT scan of the head and neck
b. MRI of the brain
c. ophthalmology review
d. ultrasound of the face

Ans: c
Ref: Dermatology, Neena Khanna, 4th edition,

Paradise Institute & Self Study Centre Page 49


PARADISE WEEKLY MODEL TEST – CEE MD/MS BASED QAE SHRAWAN 20

Port wine stains are flat vascular birthmarks present at birth. Most commonly seen on the face they rarely cross
the midline and are often dermatomal. As patients get older the port wine stain may hypertrophy and become
more cosmetically disfiguring. Treatment is with laser which is often given at the age of 1 to 2 years.
Sturge-Weber syndrome is characterised by the presence of a V1/V2 trigeminal port wine stain and one or more
of a number of abnormalities:
• ipsilateral meningeal and/or cerebral vascular malformation
• intracranial calcification
• seizures
• hemiplegia
• glaucoma
• mental retardation
• oral/lip hypertrophy.
A salmon patch is another common congenital vascular lesion. Most often seen on the forehead, glabellar and
neck they present as a dull pink-red patch with telangiectasia. Salmon patches fade with time and do not need
treatment. Fifty per cent of lesions in the nuchal region do persist but are often covered by hair.
In this question the patient has an extensive port wine stain and is likely to have Sturge-Weber syndrome. At this
age assessment of neurological involvement will not alter management and does not need to be undertaken
urgently. An early ophthalmology opinion is vital as the child may have glaucoma which would require treatment

114. “Salmon patch” or “Oil drop” sign of nail bed is seen in


a. Psoriasis
b. Lichen planus
c. Hypoalbuminemia
d. Paronychia

Ans: a
Ref: Fitzpatrick Dermatology page 215
Nail Changes in Psoriasis:
• Proximal matrix: Pitting, onychorrhexis, Beau lines
• Intermediate matrix: Leukonychia
• Distal matrix: Focal onycholysis, thinned nail plate, erythema of the lunula
• Nail bed: “Oil drop” sign or “salmon patch,” subungual hyperkeratosis, onycholysis, splinter hemorrhages
• Hyponychium: Subungual hyperkeratosis, onycholysis
• Nail plate: Crumbling and destruction plus other changes secondary to the specific site
• Proximal and lateral nail folds: Cutaneous psoriasis

115. Most common form of physical urticaria is


a. Dermographism
b. Cholinergic urticaria
c. Aquagenic urticaria
d. Solar urticaria

Ans: a
Ref: Fitzpatrick Dermatology page 418
DERMOGRAPHISM
• Dermographism is the most common form of physical urticaria and is the one most likely to be confused
with chronic urticaria. A lesion appears as a linear wheal with a flare at a site in which the skin is briskly
stroked with a firm object.

Paradise Institute & Self Study Centre Page 50


PARADISE WEEKLY MODEL TEST – CEE MD/MS BASED QAE SHRAWAN 20

• A transient wheal appears rapidly and usually fades within 30 minutes; however, the patient’s normal skin
is typically pruritic so that an itch–scratch sequence may appear. It is not associated with atopy. The peak
prevalence occurs in the second and third decades.

Anatomy

116. The radial artery lies just lateral to the tendon of which muscle?
a. Pronator teres
b. Flexor carpi radialis
c. Palmaris longus
d. Flexor carpi ulnaris

Ans: b
The radial artery lies lateral to the tendon of the flexor carpi radialis.

117. The pleuropericardial membranes develop into what structures of the adult?
a. Fibrous pericardium
b. Diaphragm
c. Parietal pleura
d. Visceral pleura

Ans: a
Ref: Moore's Anatomy 7th Edition, Page No: 131
The pleuropericardial membranes develop into the fibrous pericardium.
Development of fibrous pericardium and relocation of phrenic nerve: Exuberant growth of the lungs into the
primordial pleura cavities (pleuroperitoneal canals) cleaves the pleuropericardial folds from the body wall,
creating the pleuropericardial membranes. The membranes include the phrenic nerve and become the fibrous
pericardium that encloses the heart and separates the pleural and pericardial cavities.

118. A 3-year-old child is admitted to the ED with severe headache, high fever, malaise, and confusion.
Radiographic and physical examinations reveal that the patient suffers from meningitis. A lumbar
puncture is ordered. Which vertebral level is the most appropriate location for the lumbar puncture?
a. T12-L1
b. L1-2
c. L2-3
d. L4-5
Ans: d
Ref: Gray’s Anatomy for Students, 108
▪ A lumbar puncture is performed by taking a sample of CSF from the lumbar cistern (the subarachnoid
space below the spinal cord) between vertebrae L4 and L5 or sometimes between L3 and L4.
▪ It is done in this region because the spinal cord ends at the level of L1 to L2 and the dural sac ends at the
level of S2.
▪ Therefore it is the safest place to do the procedure because it lies between these areas and the risk of
injuring the spinal cord is avoided.
119. A 34-year-old woman is admitted to the ED after a car crash. Radiographic examination reveals a
whiplash injury in addition to hyperextension of her cervical spine. Which of the following ligaments
will most likely be injured?
a. Ligamentum flavum
b. Anterior longitudinal ligament

Paradise Institute & Self Study Centre Page 51


PARADISE WEEKLY MODEL TEST – CEE MD/MS BASED QAE SHRAWAN 20

c. Posterior longitudinal ligament


d. Anulus fibrosus

Ans: b
Ref: Gray’s Anatomy for Students, 82-86
▪ The anterior longitudinal ligament is a strong fibrous band that covers and connects the anterolateral
aspect of the vertebrae and intervertebral disks; it maintains stability and prevents hyperextension. It can
be torn by cervical hyperextension.
▪ The ligamentum flavum helps maintain upright posture by connecting the laminae of two adjacent
vertebrae.
▪ The posterior longitudinal ligament runs within the vertebral canal supporting the posterior aspect of the
vertebrae and prevents hyperflexion.
▪ The anulus fibrosus is the outer fibrous part of an intervertebral disk. The interspinous ligament connects
adjacent spinous processes.

120. On digital examination of the vagina, the portion of the uterus that one anticipates palpating with the
examining finger is the cervix and its external os. Which of the following is the most common position
of the uterus?
a. Anteflexed and retroverted
b. Retroflexed and anteverted
c. Anteflexed and anteverted
d. Retroflexed and retroverted

Ans: c
Ref: Gray’s Anatomy for Students, 455-458
Normally the uterus is anteflexed at the junction of the cervix and the body and anteverted at the junction of the
vagina and the cervical canal.

121. Which of the following segments of liver is supplied by right hepatic artery?
a. I
b. II
c. III
d. IV

Ans: a
Ref: Gray’s Anatomy, 41st edition, page 1166

122. At birth, volume of brain is what size compared to adult size?


a. 5 %
Paradise Institute & Self Study Centre Page 52
PARADISE WEEKLY MODEL TEST – CEE MD/MS BASED QAE SHRAWAN 20

b. 15 %
c. 20 %
d. 25 %

Ans: D
Ref: Gray’s Anatomy, 41st edition, page 264
• The brain of the full-term neonate ranges from 300 to 400 g in weight, with an average of 350 g; the brains of
neonatal males are slightly heavier than those of females.
• Because the head is large at birth, measuring one-quarter of the total body length, the brain is also
proportionally larger and constitutes 10% of the body weight, compared with 2% in the adult.
• At birth, the volume of the brain is 25% of its volume in adult life.
• The greater part of the increase occurs during the first year, at the end of which the volume of the brain has
increased to 75% of its adult volume. The growth can be accounted for partly by increase in the size of nerve
cell somata, the profusion and dimensions of their dendritic trees, axons and collaterals, and by the growth of
the neuroglial cells and cerebral blood vessels, but mostly it reflects the myelination of many of the axons: the
sensory pathways – visual, auditory and somatic – myelinate first, and the motor fibres later.
• During the second and subsequent years, growth proceeds much more slowly.
• The brain reaches 90% of its adult size by the fifth year and 95% by 10 years, attaining adult size by the
seventeenth or eighteenth year, largely as a result of the continuing myelination of various groups of nerve
fibres.

123. Where in the body are Meissner’s corpuscles mostly concentrated?


a. Apical tongue
b. External genitalia
c. Finger pads
d. Lips

Ans: C
Ref: Gray’s Anatomy, 41st edition, page 61
• Meissner’s corpuscles are found in the dermal papillae of all parts of the hand and foot, the anterior aspect of
the forearm, the lips, palpebral conjunctiva and mucous membrane of the apical part of the tongue.
• They are most concentrated in thick hairless skin, especially of the finger pads, where there may be up to 24
corpuscles per cm2 in young adults.
• Mature corpuscles are cylindrical in shape, approximately 80 μm long and 30 μm across, with their long axes
perpendicular to the skin surface. Each corpuscle has a connective tissue capsule and central core composed
of a stack of flat modified Schwann cells. Meissner’s corpuscles are rapidly adapting mechanoreceptors,
sensitive to shape and textural changes in exploratory and discriminatory touch; their acute sensitivity
provides the neural basis for reading Braille text.

124. Wolffian duct doesn’t give rise to which of the following structures of the testis?
a. Duct of epididymis
b. Lobules of epididymis
c. Ejaculatory duct
d. Vas deferens

Ans: B
Ref: Gray’s Anatomy, 41st edition, page 1210

Paradise Institute & Self Study Centre Page 53


PARADISE WEEKLY MODEL TEST – CEE MD/MS BASED QAE SHRAWAN 20

125. What is the sensory nerve supply of peripheral part of diaphragm?


a. Intercostal nerves
b. Long thoracic nerves
c. Phrenic nerves
d. Vagus nerve

Ans: A
Intercostal nerves
Ref: Gray’s Anatomy, 41st edition, page 973
• Sensory fibres are distributed to the peripheral part of the muscle by the lower six or seven intercostal
nerves.
• Sensory supple of central part, and motor supply is by phrenic nerve.

126. What is the origin of greater pancreatic artery?


a. Common hepatic artery
b. Gastroduodenal artery
c. Splenic artery
d. Superior mesenteric artery

Ans: C
Splenic artery
Ref: Gray’s Anatomy for Students, 4th edition, 335

Paradise Institute & Self Study Centre Page 54


PARADISE WEEKLY MODEL TEST – CEE MD/MS BASED QAE SHRAWAN 20

The arterial supply to the pancreas includes the:


o gastroduodenal artery from the common hepatic artery (a branch of the celiac trunk),
o anterior superior pancreaticoduodenal artery from the gastroduodenal artery,
o posterior superior pancreaticoduodenal artery from the gastroduodenal artery,
o dorsal pancreatic artery from the inferior pancreatic artery (a branch of the splenic artery),
o greater pancreatic artery from the inferior pancreatic artery (a branch of the splenic artery),
o anterior inferior pancreaticoduodenal artery from the inferior pancreaticoduodenal artery (a
branch of the superior mesenteric artery), and
o posterior inferior pancreaticoduodenal artery from the inferior pancreaticoduodenal artery (a
branch of the superior mesenteric artery)

127. Which visceral organ is covered by Gerota’s fascia?


a. Adrenal
b. Kidney
c. Liver
d. Spleen

Ans: B
Kidney
Ref: Gray’s Anatomy, 41stedition, page 1238
• The perirenal fascia, sometimes referred to as Gerota’s fascia, is a dense, elastic connective tissue
sheath that envelops each kidney and suprarenal gland, together with a layer of surrounding
perirenal fat. The kidney and its vessels are embedded in perirenal fat, which is thickest at the
renal borders and extends into the renal sinus at the hilum.
• The perirenal fascia was originally described as being made up of two separate entities, the
posterior fascia of Zuckerkandl and the anterior fascia of Gerota, which fused laterally to form the
lateroconal fascia.

Physiology

Paradise Institute & Self Study Centre Page 55


PARADISE WEEKLY MODEL TEST – CEE MD/MS BASED QAE SHRAWAN 20

128. A 58-year-old woman goes to her physician because she is having difficulty threading needles. An eye
examination leads to the diagnosis of presbyopia (old eyes). Her condition is most likely caused by
which of the following?
a. Ciliary muscle paralysis
b. Clouding of the vitreous
c. Retinal detachment
d. Stiffening of the lens

Ans: d
Ref: Barrett, p 188. Longo, p 224. Widmaier, pp 204-205.)
The increase in lens power that normally occurs when objects are placed close to the eye (the accommodation
reflex) does not take place in presbyopia. The failure of the accommodation reflex occurs because the lens and
lens capsule stiffen with age. There are some reports of ciliary muscle weakness accompanying presbyopia, but
there are none indicating that presbyopia is caused by ciliary muscle paralysis.

129. A 16-year-old, highly allergic girl who is stung by a bee gives was given epinephrineat ED. Because
epinephrine activates β- adrenergic receptors, it will relieve the effects of the bee sting by decreasing
which of the following?
a. Contraction of airway smooth muscle
b. Rate of depolarization in the SA node
c. Rate of glycogenolysis in the liver
d. Transport of calcium into skeletal muscle fibers

Ans: a
Ref: Barrett, pp 145-147, 357-358, 447. Longo, pp 2709-2710.)
(Epinephrine (adrenalin) acts on both α-and β-adrenergic receptors, but has a greater affinity for β-adrenergic
receptors. Activation of β2-adrenergic receptors leads to relaxation of smooth muscle in the bronchi, vasculature,
intestine, uterus, and bladder; to increased pancreatic insulin and glucagon secretion; and an increase in liver
glycogenolysis. The bronchodilator effects of epinephrine are key in the treatment of the life-threatening effects
of anaphylactic shock. Activation of β1- and β-2-adrenergic receptors in the heart leads to an increase in the rate
of SA nodal phase 4 depolarization and thus heart rate (positive chronotropic response), an increase in
contractility (positive inotropic response), an increase in conduction velocity
(positive dromotropic response), and an increase in cardiac excitability/irritability. The transport of Ca2+ into
skeletal muscle fibers is not affected by β-receptors. The effects of epinephrine-induced β-adrenergic receptor
activation are due to G-protein–mediated activation of adenylate cyclase, which catalyzes the formation of cyclic
adenosine monophosphate and activation of protein kinase A.

130. Cardiac index is defined as:


a. Stroke volume/m2 BSA
b. CO per unit body surface area
c. Systolic pressure/m2 BSA
d. End diastolic volume

Ans: b
Ref: Ganong physiology 25th Edition, Page No: 544
The amount of blood pumped out of the heart per beat, the stroke volume, is about 70 mL from each ventricle in
a resting man of average size in the supine position. The output of the heart per unit of time is the cardiac output.
In a resting, supine man, it averages about 5.0 L/min (70 mL × 72 beats/min). There is a correlation between

Paradise Institute & Self Study Centre Page 56


PARADISE WEEKLY MODEL TEST – CEE MD/MS BASED QAE SHRAWAN 20

resting cardiac output and body surface area. The output per minute per square meter of body surface (the cardiac
index) averages 3.2 L

131. Meissner’s plexus is located in


a. Between the mucosa and inner circular muscle layer
b. Between inner circular layer and outer longitudinal layer
c. Between outer longitudinal layer and serosa
d. Mucosa

Ans: a
Ref: Ganong physiology 25th Edition, Page No: 472
• Two major networks of nerve fibers are intrinsic to the gastrointestinal tract: the myenteric plexus
(Auerbach plexus), between the outer longitudinal and middle circular muscle layers, and the submucous
plexus (Meissner plexus), between the middle circular layer and the mucosa.
• Collectively, these neurons constitute the enteric nervous system. The system contains about 100 million
sensory neurons, interneurons, and motor neurons in humans—as many as are found in the whole spinal
cord—and the system is probably best viewed as a displaced part of the central nervous system (CNS) that
is concerned with the regulation of gastrointestinal function.
• It is sometimes referred to as the “little brain” for this reason. It is connected to the CNS by
parasympathetic and sympathetic fibers but can function autonomously without these connections.
The myenteric plexus innervates the longitudinal and circular smooth muscle layers and is concerned primarily
with motor control, whereas the submucous plexus innervates the glandular epithelium, intestinal endocrine cells,
and submucosal blood vessels and is primarily involved in the control of intestinal secretion.

132. Hypertonic urine is excreted due to absorption of water in


a. Collecting duct
b. PCT
c. Ascending loop
d. Descending loop

Ans: a
REF: Ganong physiology,25thedition, page 684-685 / Guyton 13th edition page 374
• The final concentration of urine is determined by the amount of water reabsorption that occurs in the
collecting ducts. The change in osmolarity of the urine depends on the amount of ADH acting on the
collecting ducts.
• The basic requirement for concentrated urine are: High level of ADH and high osmolarity of the renal
medullary interstitial fluid which provides the osmotic gradient necessary for water reabsorption to occur
in the presence of high level of ADH.

133. Insulin resistance down regulates which receptor


a. GLUT 1
b. GLUT 2
c. GLUT 3
d. GLUT 4

Ans: d
Ref: Ganong physiology 25th Edition, Page No: 433
GLUT-4 is an insulin dependent GLUT. Other GLUTs are not insulin sensitive and are constitutively expressed
in the cell membrane. Thus GLUT 4 is downregulated in case of insulin resistance.

Paradise Institute & Self Study Centre Page 57


PARADISE WEEKLY MODEL TEST – CEE MD/MS BASED QAE SHRAWAN 20

Table: Glucose transporters

134. A 42-year-old salesman presents with the chief complaint of intermittent midepigastric pain that is
relieved by antacids or eating. Gastric analysis reveals that basal and maximal acid outputs exceed
normal values. The gastric acid hypersecretion can be explained by an increase in the plasma
concentration of which of the following?
a. Cholecystokinin
b. Gastrin
c. Secretin
d. Somatostatin

Ans :b
Ref: (Barrett, pp 457-461. Kaufman, pp 114-117. Le, pp 319-320, 327.
Increases in basal and maximal acid output are suggestive of inflammation or removal of the proximal small
intestine. Intestinal receptors monitor the composition of chyme and elicit feedback mechanisms that regulate
gastric acid secretion and gastric emptying. The absence of feedback leads to an increased presence of excitatory
mediators of gastric function. Gastrin is the primary stimulus of meal-induced acid secretion by the parietal cells.
Somatostatin, vasoactive intestinal peptide, secretin, and cholecystokinin inhibit gastric acid secretion.

135. What is the normal pleural pressure at the beginning of inspiration?


a. 5 cm of water
b. 0 cm of water
c. -5 cm of water
d. -7.5 cm of water

Ans: C
-5cm of water
Ref: Guyton & Hall Physiology, 13th edition, page 498
• Pleural pressure is the pressure of the fluid in the thin space between the lung pleura and the chest wall pleura.
As noted earlier, this pressure is normally a slight suction, which means a slightly negative pressure.
• The normal pleural pressure at the beginning of inspiration is about −5 centimeters of water, which is the
amount of suction required to hold the lungs open to their resting level.
During normal inspiration, expansion of the chest cage pulls outward on the lungs with greater force and creates
more negative pressure, to an average of about −7.5 centimeters of water.

Paradise Institute & Self Study Centre Page 58


PARADISE WEEKLY MODEL TEST – CEE MD/MS BASED QAE SHRAWAN 20

136. A 39-year-old man with an enlarged head, hands, and feet; osteoarthritic vertebral changes; and
hirsutism presents with a complaint of gynecomastia and lactation. The patient is most likely suffering
from a tumor in which of the following locations?
a. Adrenal cortex
b. Anterior pituitary
c. Hypothalamus
d. Posterior pituitary

Ans: B
Ref: (Barrett, pp 380-389. Kacsoh, pp 151-161. Kaufman, pp 74-75, 233.)
• Tumors of the somatotropes of the anterior pituitary gland secrete large amounts of growth hormone,
leading to acromegaly in adults. When the epiphyses have not yet fused to the long bones, growth is
stimulated by excess growth hormone leading to gigantism in children. Once the epiphyses have closed,
linear growth is no longer possible, and growth hormone produces the pattern of bone and soft-tissue
abnormalities typical of acromegaly.
• Hypersecretion of growth hormone is accompanied by hypersecretion of prolactin in up to 40% of patients
with acromegaly, explaining the patient’s complaint of lactation. Human growth hormone also has
intrinsic lactogenic activity, contributing to the development of gynecomastia and lactation in this male
patient. Acromegaly can be caused by hypothalamic tumors that secrete growth hormone–releasing
hormone (GHRH), but these are rare.
• Posterior pituitary releases oxytocin and ADH, which are not involved in this case. Adrenal cortex tumors
release corticotropins leading to Cushing’s syndrome, primary aldosteronism, or rarely virilization.

137. Radiation treatment for a pituitary tumor in an 8-year-old boy results in complete loss of pituitary
function. As a result, the child is likely to experience which of the following symptoms?
a. Accelerated growth spurts
b. Hyporeflexia
c. Hyperactivity
d. Sexual precocity

Ans: B
Ref: (Barrett, pp 396-397, 404-409. Kaufman, pp 72-76. Le, p 325.)
Radiation treatment likely produces panhypopituitarism in the young child. Sexual maturation and growth during
development will not occur because of low levels of growth hormone, FSH, LH, IGF-I, TSH and thyroid
hormones, and gonadal hormones. The cortisol response to stress is decreased due to low ACTH levels. The
decrease in TSH and thyroid hormones causes hypoactivity, lethargy, fatigue, weakness, and decreased reflexes.

138. An 18-year-old man with hemophilia A suffered multiple internal injuries from a accident. He is now
presenting with dizziness, abdominal pain, dark patches on his elbows and knees, and cravings for food.
He is referred to an endocrinologist who makes the diagnosis of Addison’s disease, and prescribes
cortisol. Cortisol administration to a patient with adrenal insufficiency will result in which of the
following?
a. Enhanced wound healing
b. Increased ACTH secretion
c. Increased corticotropin-releasing hormone (CRH) secretion
d. Increased gluconeogenesis

Ans: D

Paradise Institute & Self Study Centre Page 59


PARADISE WEEKLY MODEL TEST – CEE MD/MS BASED QAE SHRAWAN 20

Ref: (Barrett, pp 366-372. Kaufman, pp 89-91. Widmaier, p 344.)


Cortisol is defined as a glucocorticoid because it promotes the conversion of amino acids to glucose
(gluconeogenesis). It also decreases glucose uptake by muscle and adipocytes by decreasing the sensitivity of the
cells to insulin. The net result is to provide more glucose to non–insulin requiring cells. Cortisol retards wound
healing. It also decreases CRH and ACTH secretion by feedback inhibition

139. A sarcomere is an area between which of the following structures lying adjacent?
a. H band
b. I band
c. M line
d. Z line

Ans: D
Z line
Ref: Ganong’s Review of Physiology, 25th edition, page 101

• Differences in the refractive indexes of the various parts of the muscle fiber are responsible for the
characteristic cross-striations seen in skeletal muscle when viewed under themicroscope. The parts of
the cross-striations are frequently identified by letters. The light I band is divided by the dark Z line,
and the dark A band has the lighter H band in its center. A transverse M line is seen in the middle of
the H band, and this line plus the narrow light areas on either side of it are sometimes called the
pseudo-H zone.
• The area between two adjacent Z lines is called a sarcomere. The orderly arrangement of actin,
myosin, and related proteins that produces this pattern.
• The thick filaments, which are about twice the diameter of the thin filaments, are made up of myosin;
the thin filaments are made up of actin, tropomyosin, and troponin. The thick filaments are lined up to
form the A bands, whereas the array of thinfilaments extends out of the A band and into the less dense
staining I bands. The lighter H bands in the center of the A bands are the regions where, when the
muscle is relaxed, the thin filaments do not overlap the thick filaments.

Paradise Institute & Self Study Centre Page 60


PARADISE WEEKLY MODEL TEST – CEE MD/MS BASED QAE SHRAWAN 20

Pathology

140. Earliesttransient change following tissue injury will be:


a. Neutropenia
b. Neutrophilia
c. Monocytosis
d. Lymphocytosis

Ans: b
Ref: Robbins Basic Pathology 10th Edition, Page No: 65
The nature of the leukocyte infiltrate varies with the age of the inflammatory response and the type of stimulus.
In most forms of acute inflammation, neutrophils predominate in the inflammatory infiltrate during the first 6
to 24 hours and are gradually replaced by monocyte-derived macrophages over 24 to 48 hours.

141. ‘Popcorn cells’ are seen in which type of Hodgkin’s disease?


a. Lymphocyte dominant
b. Lymphocyte depleted
c. Nodular sclerosis
d. Mixed type

Ans: a
Ref: Robbins Basic Pathology 10th Edition, Page No: 475
Lymphocyte-predominant Hodgkin lymphoma, accounting for about 5% of cases, is characterized by the
presence of lymphohistiocytic (L&H) variant RS cells that have a delicate multilobed, puffy nucleus resembling
popped corn (“popcorn cell”).
▪ Lacunar cell is a variant of RS cell having a clear space surrounding the cell.
▪ Popcorn cell is a lymphohistiocytic (L-H) variant of RS cell having a multilobed nucleus resembling a
popcorn kernel.
▪ CD 30 is the most sensitive marker of Reed Sternberg cell.
▪ Cells similar or identical in appearance to Reed-Sternberg cells are also seen in other conditions like
infectious mononucleosis, solid tissue cancers, and NHL (Immunoblastic lymphoma).
▪ Thus, although Reed-Sternberg cells are requisite for the diagnosis, they must be present in an appropriate
background of non-neoplastic inflammatory cells (lymphocytes, plasma cells, eosinophils).

142. Gene for Wilms’ tumour is located on which of the following?


a. Chromosome 1
b. Chromosome 10
c. Chromosome 11
d. Chromosome 12

Ans: c
Ref: Robbins Basic Pathology 10th Edition, Page No: 289
Three groups of congenital malformations are associated with an increased risk for Wilms tumor.
These are WAGR syndrome (i.e., Wilms tumor, aniridia, genital abnormalities, and mental retardation); Denys-
Drash syndrome (DDS), and Beckwith- Wiedemann syndrome (BWS).
Of patients with WAGR syndrome approximately one in three will go on to develop this tumor. Another group of
patients, those with so-called “Denys-Drash syndrome” (DDS), have an even higher risk (approximately 90%) of
Wilms tumor. Patients with the WAGR syndrome, as the name indicates, have Wilms tumor, aniridia, genital
abnormalities and mental retardation.

Paradise Institute & Self Study Centre Page 61


PARADISE WEEKLY MODEL TEST – CEE MD/MS BASED QAE SHRAWAN 20

DD syndrome is characterized by gonadal dysgenesis and early onset nephropathy leading to renal failure.
Both of these conditions are associated with abnormalities of the Wilms tumor 1 (WT1) gene, located on
11p13.

143. What is the commonest cause of hypertension?


a. Adrenocortical hyperfunction
b. Coarctation of aorta
c. Essential
d. Renal artery stenosis

Ans: C
Ref: Robbins Basic Pathology, 10th edition, page 367

144. What is not a feature of tetralogy of Fallot?


a. Left ventricular hypertrophy
b. Overriding of the VSD by the aorta
c. Subpulmonic stenosis
d. Ventricular septal defect

Ans: A
Ref: Robbins Basic Pathology, 10th edition, page 406
• Tetralogy of Fallot is the most common cause of cyanotic congenital heart disease. It accounts for about 5%
of all congenital cardiac malformations. The four cardinal features are:
▪ VSD
▪ Right ventricular outflow tract obstruction (subpulmonic stenosis)

Paradise Institute & Self Study Centre Page 62


PARADISE WEEKLY MODEL TEST – CEE MD/MS BASED QAE SHRAWAN 20

▪ Overriding of the VSD by the aorta


▪ Right ventricular hypertrophy
• All of the features of tetralogy of Fallot result from anterosuperior displacement of the infundibular septum
leading to abnormal septation between the pulmonary trunk and the aortic root.

145. What is cingulate herniation also known as?


a. Subfalcine herniation
b. Transtentorial herniation
c. Tonsillar herniation
d. Uncal herniation

Robbins Basic Pathology, 10th edition, page 851 – 852


There are three main types of herniation:
• Subfalcine (cingulate) herniation occurs when unilateral or asymmetric expansion of a cerebral hemisphere
displaces the cingulate gyrus under the edge of the falx. This may compress the anterior cerebral artery.
• Transtentorial (uncinate) herniation occurs when the medial aspect of the temporal lobe is compressed
against the free margin of the tentorium. As the temporal lobe is displaced, the third cranial nerve is
compromised, resulting in pupillary dilation and impaired ocular movements on the side of the lesion (“blown
pupil”). The posterior cerebral artery may also be compressed, resulting in ischemic injury to tissue supplied
by that vessel, including the primary visual cortex. With further displacement of the temporal lobe, pressure
on the midbrain may compress the contralateral cerebral peduncle against the tentorium, resulting in
hemiparesis ipsilateral to the side of the herniation. The compression of the peduncle creates a deformation
known as Kernohan’s notch. Compression of the midbrain and the ascending reticular activating system with
transtentorial herniation leads to depressed consciousness. Progression of transtentorial herniation is often
accompanied by linear or flame-shaped hemorrhages in the midbrain and pons, termed Duret hemorrhages.
These lesions usually occur in the midline and paramedian regions and are believed to be the result of tearing
of penetrating veins and arteries supplying the upper brain stem.
• Tonsillar herniation refers to displacement of the cerebellar tonsils through the foramen magnum. This type
of herniation causes brain stem compression and compromises vital respiratory and cardiac centers in the
medulla, and is often fatal.

146. What is the most common antigenic target in Myasthenia gravis?

Paradise Institute & Self Study Centre Page 63


PARADISE WEEKLY MODEL TEST – CEE MD/MS BASED QAE SHRAWAN 20

a. Presynaptic acetylcholine receptor


b. Presynaptic noradrenaline receptor
c. Postsynaptic acetylcholine receptor
d. Postsynaptic noradrenaline receptor

Ans: C
Ref: Robbins Basic Pathology, 10th edition, page 839
• Myasthenia gravis is an autoimmune disease with fluctuating muscle weakness that is caused by
autoantibodies that target the neuromuscular junction.
• The most common antigenic target is the postsynaptic acetylcholine receptor (AChR). Other
pathogenic antibodies recognize muscle-specific kinase (MuSK) and low-density lipoprotein
receptor-related protein (LRP4). These antibodies lead to loss of receptors and damage to the
structure of the junctions.

147. What is the translocation seen in Ewing’s sarcoma?


a. 11;22
b. 10;21
c. 9;12
d. 7;21

Ans: A
Ref: Robbins Basic Pathology, 10th edition, page 813
• The vast majority (85%) of Ewing sarcomas contain a balanced (11;22) (q24;q12) translocation generating in-
frame fusion of the EWSR1 gene on chromosome 22 to the FLI1 gene on chromosome 11.
• Variant translocations fuse EWSR1 to other members of the ETS transcription factor family. How EWS
fusion proteins contribute to transformation remains unsettled.

148. Which of the following gene abnormality is rare in small cell carcinoma of lung?
a. 3p deletions
b. KRAS mutations
c. RB mutations
d. TP53 mutations

Ans: B
Ref: Robbins Basic Pathology, 10th edition, page 542

Paradise Institute & Self Study Centre Page 64


PARADISE WEEKLY MODEL TEST – CEE MD/MS BASED QAE SHRAWAN 20

149. Which of the following is most commonly associated with granulomatosis with polyangiitis?
a. Bilateral pneumonitis with nodules and cavitary lesions
b. Chronic sinusitis
c. Mucosal erosions
d. Renal diseases

Ans: A
Ref: Robbins Basic Pathology, 10th edition, page 388
• Previously called Wegener granulomatosis, granulomatosis with polyangiitis (GPA) is a necrotizing vasculitis
characterized by a triad of the following:
o Necrotizing granulomas of the upper-respiratory tract (ear, nose, sinuses, throat) or the lower-
respiratory tract (lung) or both
o Necrotizing or granulomatous vasculitis affecting small- to medium-sized vessels (e.g., capillaries,
venules, arterioles, and arteries), most prominently the lungs and upper airways but other sites as well
o Focal necrotizing, often crescentic, glomerulonephritis
• The typical patient is a middle aged man, although women and individuals of other ages can be affected.
Classic presentations include bilateral pneumonitis with nodules and cavitary lesions (95%), chronic sinusitis
(90%), mucosal ulcerations of the nasopharynx (75%), and renal disease (80%).

Pharmacology

150. Where is cortisol mostly metabolized?


a. Kidney
b. Liver
c. Lungs
d. Wall of intestines

Paradise Institute & Self Study Centre Page 65


PARADISE WEEKLY MODEL TEST – CEE MD/MS BASED QAE SHRAWAN 20

Ans: B
Ref: Katzung Pharmacology, 14th edition, page 705
• The half-life of cortisol in the circulation is normally about60–90 minutes; it may be increased when
hydrocortisone (the pharmaceutical preparation of cortisol) is administered in large amountsor when stress,
hypothyroidism, or liver disease is present.
• Only 1%of cortisol is excreted unchanged in the urine as free cortisol; about20% of cortisol is converted to
cortisone by 11-hydroxysteroiddehydrogenase in the kidney and other tissues with mineralocorticoid
receptors before reaching the liver.
• Most cortisol is metabolized in the liver. About one-third of the cortisol produced daily is excreted in the
urine as dihydroxy ketone metabolites and is measured as 17-hydroxysteroids.
Many cortisol metabolites are conjugated with glucuronic acid or sulfate at the C3 and C21 hydroxyls,
respectively, in the liver; they are then excreted in the urine

151. Which is not an alkylating agent?


a. Cyclophosphamide
b. Busulphan
c. 5-Fluorouracil
d. Melphalan

Ans: c
Ref: KDT Pharmacology 7th Edition, Page No: 858
Alkylating agents
Nitrogen mustards
Mechlorethamine (Mustine HCl) Cyclophosphamide, Ifosfamide, Chlorambucil, Melphalan
Ethylenimine
Thio-TEPA
Alkyl sulfonate
Busulfan
Nitrosoureas
Carmustine , Lomustine
Triazine
Dacarbazine, Temozolomide
Methyl hydrazine
Procarbazine
5-Flourouracil-Antimetabolites (pyrimidine antagonist)

152. Which Of the following is not adverse effect of imatinib?


a. Periorbital edema
b. Myalgia
c. Pleural effusion
d. Arthralgia

Ans: d
Ref: KDT Pharmacology 7th Edition, Page No: 870
• Imatinib -It is the first selectively targeted drug to be introduced for treatment of a malignancy. It inhibits
a specific tyrosine protein kinase labelled ‘Bcr-Abl’ tyrosine kinase expressed by chronic myeloid
leukaemia (CML) cells and related receptor tyrosine kinases including platelet derived growth factor
(PDGF) receptor.

Paradise Institute & Self Study Centre Page 66


PARADISE WEEKLY MODEL TEST – CEE MD/MS BASED QAE SHRAWAN 20

• Imatinib is found to be strickingly successful in chronic phase of CML (remission in >90% cases) and in
metastatic c-kit-positive GIST, in which it is the drug of choice.
Adverse effects are
• abdominal pain, vomiting, fluid retention, periorbital edema, pleural effusion, myalgia, liver
damage and CHF

153. Insulin secretion increasing drug by acting on beta cells of pancreas is:
a. Metformin
b. Repaglinide
c. Pioglitazone
d. Acarbose

Ans: b
Ref: KDT Pharmacology 7th Edition, Page No: 271-277
Repaglinide-This meglitinide analogue oral hypoglycemic is designed to normalize meal- time glucose
excursions. Though not a sulfonyl- urea, it acts in an analogous manner by binding to sulfonyl- urea receptor→
closure of ATP dependent K+ channels → depolarization → insulin release.
Metformin reduce blood glucose
1. Suppresses hepatic gluconeogenesis and glucose output from liver.
2. Enhances insulin-mediated glucose uptake and disposal in skeletal muscle and fat. Insulin resistance
exhibited by type-2 diabetics is thus overcome.
3. Interferes with mitochondrial respiratory chain and promotes peripheral glucose utilization through
anaerobic glycolysis.
Pioglitazone-This class of oral antidiabetic drugs are selective agonists for the nuclear Peroxisome proliferator-
activated receptor γ (PPARγ) which is expressed mainly in fat cells, but also in muscle and some other cells. It
enhances the transcription of several insulin responsive genes. Glitazones tend to reverse insulin resistance by
enhancing GLUT4 expression and translocation. Entry of glucose into muscle and fat is improved. Hepatic
gluconeogenesis is also suppressed. Activation of genes regulating fatty acid metabolism and lipogenesis in
adipose tissue contributes to the insulin sensitizing action.
Acarbose-It is a complex Oligosaccharide Which reversibly inhibits α-glucosidases, the final enzymes for the
digestion of carbohydrates in the brush border of small intestine mucosa. It slows down and decreases digestion
and absorption of polysaccharides (starch, etc.) and sucrose.

154. What agent is used to reverse the biological effects of methotrexate in an accidental overdose
scenario?
a. Folic acid
b. Leucovorin
c. Platelets
d. Vitamin K

Ans: B
Ref:Katzung Pharmacology, 14th edition, page 957
• The biologic effects of MTX can be reversed by administration of the reduced folate leucovorin (5-
formyltetrahydrofolate) or by L-leucovorin, which is the active enantiomer. Leucovorin rescue is used in
conjunction with high-dose MTX therapy to rescue normal cells from undue toxicity, and it has also been
used in cases of accidental drug overdose.

155. Which among the following is also known as Vitamin K3?


a. Menadione

Paradise Institute & Self Study Centre Page 67


PARADISE WEEKLY MODEL TEST – CEE MD/MS BASED QAE SHRAWAN 20

b. Menaquinone
c. Phylloquinone
d. Phytonadione

Ans: A
Ref: KDT Pharmacology, 8th edition, page 972
The available forms of Vitamin K are:
• Phytonadione (K1) (Phylloquinone)
• Menaquinone (K2)
• Menadione (K3)
• Acetomenapthone

156. What is not a mechanism of resistance of aminoglycoside?


a. Alteration or deletion of receptor protein on the 30S ribosomal subunit
b. Efflux pumps
c. Impaired entry of aminoglycoside into the cell
d. Inactivation of aminoglycoside by transferase enzyme

Ans: B
Ref: Katzung Pharmacology, 14th edition, page 827 – 828
• Three principal mechanisms of resistance have been established:
o production of a transferase enzyme that inactivates the aminoglycoside by adenylylation,
acetylation, or phosphorylation. This is the principal type of resistance encountered clinically.
o There is impaired entry of aminoglycoside into the cell. This may result from mutation or deletion
of a porin protein involved in transport and maintenance of the electrochemical gradient or from
growth conditions under which the oxygen-dependent transport process is not functional.
o The receptor protein on the 30S ribosomal subunit may be deleted or altered as a result of a
mutation.

157. Which of the following is inducible nitric oxide synthase (iNOS)?


a. NOS-1
b. NOS-2
c. NOS-3
d. NOS-4

Ans: b
Ref: KD Tripathi Essentials of Medical Pharmacology; 7th Edition, Page No: 151
Carvedilol: It is a β1 + β2 + α1 adrenoceptor blocker; produces vasodilatation due to α1 blockade as well as
calcium channel blockade, and has antioxidant property. It has been used in hypertension and is the β blocker
especially employed as cardioprotective in CHF.

158. Which cells of pancreas secretes ghrelin?


a. Alpha
b. Beta
c. Delta
d. Epsilon

Ans: D, Epsilon
Ref: Katzung Pharmacology, 14th edition, page 748

Paradise Institute & Self Study Centre Page 68


PARADISE WEEKLY MODEL TEST – CEE MD/MS BASED QAE SHRAWAN 20

159. What is the most common adverse effect of thioamides?


a. Exfoliative dermatitis
b. Lupus like reaction
c. Maculopapular pruritic rash
d. Urticarial rash

Ans: C
Ref: Katzung Pharmacology, 14th edition, page 695
• Adverse reactions to the thioamides occur in 3–12% of treated patients. Most reactions occur early, especially
nausea and gastrointestinal distress. An altered sense of taste or smell may occur with methimazole.
• The most common adverse effect is a maculopapular pruritic rash (4–6%), at times accompanied by systemic
signs such as fever. Rare adverse effects include an urticarial rash, vasculitis, a lupus-like reaction,
lymphadenopathy, hypoprothrombinemia, exfoliative dermatitis, polyserositis, and acute arthralgia.
• An increased risk of severe hepatitis, sometimes resulting in death, has been reported with propylthiouracil
(black box warning), so it should be avoided in children and adults unless no other options are available.
Cholestatic jaundice is more common with methimazole than propylthiouracil. Asymptomatic elevations in
transaminase levels can also occur.
• The most dangerous complication is agranulocytosis (granulocyte count < 500 cells/mm3), an infrequent but
potentially fatal adverse reaction. It occurs in 0.1–0.5% of patients taking thioamides, but the risk may be
increased in older patients and usually within the first 90 days in those receiving more than 40 mg/dof
methimazole. The reaction is usually rapidly reversible when the drug is discontinued, but broad-spectrum
antibiotic therapy may be necessary for complicating infections. Colony-stimulating factors may hasten
recovery of the granulocytes. The cross-sensitivity between propylthiouracil and methimazole is about 50%;
therefore, switching drugs in patients with severe reactions is not recommended.

Biochemistry

160. (CAG)n trinucleotide repeat is seen in:


a. Huntington disease
b. Myotonic dystrophy
c. Fragile X syndrome
d. Friedreich ataxia

Ans: a
Ref: Harper Biochemistry 30th Edition, Page No: 378
Trinucleotide sequences that increase in number (microsatellite instability) can cause disease.

Paradise Institute & Self Study Centre Page 69


PARADISE WEEKLY MODEL TEST – CEE MD/MS BASED QAE SHRAWAN 20

Diseases Trinucleotide repeat


Fragile X syndrome (CGG)
Huntington’s chorea (CAG)
Myotonic dystrophy (CTG)
Spinobulbar muscular atrophy (CAG)
Kennedy disease (CAG)

Friedreich ataxia (GAA)

161. Porphyria cutanea tarda is due to deficiency of which enzyme:


a. Uroporphyrinogen I synthase
b. Uroporphyrinogen decarboxylase
c. Coproporphyrinogen oxidase
d. Protoporphyrinogen oxidase

Ans: b
Ref: Satyanarayan Biochemistry, 4th edition, Page No: 203
Porphyria Enzyme deficiency

Acute intermittent porphyria Uroporphyrinogen I synthase


Porphyria cutanea tarda Uroporphyrinogen
decarboxylase
Hereditary coproporphyria Corpoporphyrinogen oxidase
Variegate porphyria Protoporphyrinogen oxidase
Congenital erythropoietic Uroporphyrinogen III cosynthase
porphyria
Protoporphyria Ferrochelatase

162. Insulin-dependent transport system occurs in:


a. Hepatocytes
b. Erythrocytes
c. Brain
d. Muscle

Ans: d
Ref: Satyanarayan Biochemistry, 4th edition, Page No: 245
Entry of glucose into cells
Glucose concentration is very low in the cells compared to plasma (for humans < 100 mg/dl).However, glucose
does not enter the cells by simple diffusion. Two specific transport systems are recognized for the entry of
glucose into the cells.

Paradise Institute & Self Study Centre Page 70


PARADISE WEEKLY MODEL TEST – CEE MD/MS BASED QAE SHRAWAN 20

1. Insulin-independent transport system of glucose : This is a carrier mediated uptake of glucose which is not
dependent on the hormone insulin. This is operative in hepatocytes, erythrocytes and brain.
2. Insulin-dependent transport system: This occurs in muscle and adipose tissue.

163. Which complex of electron transport chain is inhibited by cyanide:


a. Complex I
b. Complex II
c. Complex III
d. Complex IV
Ans: d
Ref: Harper Biochemistry 30th Edition, Page No: 132
Barbiturates such as amobarbital inhibit electron transport via
• Complex I by blocking the transfer from Fe-S to Q.
• Malonate is a competitive inhibitor of Complex II.
• Antimycin A and dimercaprol inhibit the respiratory chain at Complex III.
• H2S, carbon monoxide, and cyanide inhibit Complex IV
Atractyloside inhibits oxidative Phosphorylation by inhibiting the transporter of ADP into and ATP out of the
mitochondrion The antibiotic oligomycin completely blocks oxidation and Phosphorylation by blocking the flow
of protons through ATP synthase.

164. What is the geometric relationship between a given set of atoms known as?
a. Configuration
b. Conformation
c. Isomerization
d. Translation

Ans: A
Configuration
Ref: Harper’s Biochemistry, 30th edition, page 36
• Configuration refers to the geometric relationship between a given set of atoms, for example, those
that distinguish l- from d-amino acids. Interconversion of configurational alternatives requires
breaking (and reforming) covalent bonds.
• Conformation refers to the spatial relationship of every atom in a molecule. Interconversion
between conformers occurs with retention of configuration, generally via rotation about single
bonds.

165. Which of the following posttranslational modification causes maximum increase in mass?
a. Acetylation
b. Glycosylation
c. Methylation
d. Phosphorylation

Ans: B
Ref: Harper’s Biochemistry, 30th edition, page 31

Paradise Institute & Self Study Centre Page 71


PARADISE WEEKLY MODEL TEST – CEE MD/MS BASED QAE SHRAWAN 20

166. Which of the following is not a hydrophobic amino acid?


a. Arginine
b. Leucine
c. Methionine
d. Valine

Ans: A, Arginine
Ref: Harper’s Biochemistry, 30th edition, page 18

167. Which of the following enzymes in TCA cycle is associated with production of FADH2?
a. Fumarase
b. Isocitrate dehydrogenase
c. Succinate dehydrogenase
d. Succinate thiokinase

Ans: C, Succinate dehydrogenase


Ref: Harper’s Biochemistry, 30th edition, page 163

Paradise Institute & Self Study Centre Page 72


PARADISE WEEKLY MODEL TEST – CEE MD/MS BASED QAE SHRAWAN 20

Microbiology

168. A 15-year-old boy is taken to his pediatrician after experiencing fever, malaise, and anorexia followed
by tender swelling of his parotid glands. Which of the following is the most likely complication to occur in
this patient?
a. Guillain–Barré syndrome
b. Hemorrhage
c. Oophoritis
d. Orchitis

Ans: d
Ref: Levinson, Ch 39, 46. Murray, Ch 56. Ryan, Ch 9, 10.)
The most common complication of mumps virus infection in postpubertal males is orchitis (e), occurring in about
33% to 50% of cases. Other complications occur at lower frequencies including myocarditis,
meningoencephalitis, Guillain–Barré syndrome , mastitis, pneumonia, thyroiditis, pancreatitis, hearing loss, and
immune thrombocytopenia purpura, which may be associated with hemorrhage. Oophoritis occurs in
approximately 7% of postpubertal females.

169. Which of the following parasite doesn’t enter into body by skin penetration?
a. Dracunculus
b. Necator Americana

Paradise Institute & Self Study Centre Page 73


PARADISE WEEKLY MODEL TEST – CEE MD/MS BASED QAE SHRAWAN 20

c. Ancylostoma duodenale
d. Strongyloides

Ans: a
Ref: Jawetz Microbiology 26th Edition, Page No: 744
The guinea worm, Dracunculus medinensis, has an aquatic cycle via copepods (“water fleas”—an abundant group
of aquatic microcrustaceans). Copepods ingest larvae released from human skin blisters that burst when immersed
in cold water, spewing great numbers of larvae. Infected copepods are inadvertently ingested by drinking
unfiltered, infested water. After a year of systemic wandering in the body, the worms mature and mate. The
females then travel to the skin, usually to the lower leg—where they induce blisters that form near the foot and
ankle.
Ancylostoma duodenal and Necatar Americanus(Human Hookworm-Intestinal nematode)
Larvae penetrate host skin and migrate throughout the host similarly to Ascaris and end up in the small intestine
where they mature into adult worms.

170. An outbreak of hepatitis occurred in an area with poor sanitation. Most of the patients reported fever,
nausea with vomiting, and weight loss occurring over several days followed by jaundice and pruritus.
Testing quickly ruled out HAV. A number of women in the area are pregnant. For which of the following
are these women at risk?
a. Chronic hepatitis
b. Fetal hydrops
c. Fulminant hepatic failure
d. Guillain–Barré syndrome

Ans: c
Ref: (Andary. Levinson, Ch 41. Murray, Ch 63. Ryan, Ch 13. Weiner.)
Recognized in the 1980s, HEV, Hepevirus, is a ss(+)RNA nonenveloped virus placed in a new family—
Hepeviridae. The virus is found in Southeast and Central Asia, Africa, North America (Mexico), South America,
and Australia. Epidemics have been observed in Asia, Africa, India, and Mexico. Like HAV, it is enterically
transmitted, often by water contaminated with human feces. Epidemics frequently affect young adults more than
those of other ages. The overall fatality rate is 4%, but pregnant women more frequently develop fulminant
hepatic failure and 20% die from complications. Chronic hepatitis (a) does not occur in normal individuals in
whom the infection resolves; there is association of HEV with chronic hepatitis in persons who already have
hepatitis, liver transplantation, or who are immunocompromised in humoral immune responses. Fetal hydrops
may occur in women who acquire parvovirus B19 infection during pregnancy; Guillain–Barré syndrome is a
cluster of clinical syndromes that include acute inflammatory demyelinating polyradiculoneuropathy, acute motor
axonal neuropathy, and acute motor-sensory axonal neuropathy that are associated with a number of
microorganisms: Campylobacter
jejuni, CMV, EBV, Mycoplasma pneumoniae, VZV, and HIV.
Reye syndrome, acute noninflammatory encephalopathy and fatty degenerative liver failure, occurs primarily in
children after illness with influenza A or B virus or VZV, especially in conjunction with use of aspirin to reduce
fever. Numerous other viruses and some bacteria have also been implicated. However, the incidence of Reye
syndrome has declined with the decrease in aspirin usage in children.

171. Which of the following cells if responsible for virus reproduction in HIV?
a. CD4 T lymphocytes
b. CD8 T lymphocytes
c. NK cells
d. Monocytes

Paradise Institute & Self Study Centre Page 74


PARADISE WEEKLY MODEL TEST – CEE MD/MS BASED QAE SHRAWAN 20

Ans: A
Ref: Jawetz Microbiology, 27th edition, page 643
• An immune response to HIV occurs 1 week to 3 months after infection, plasma viremia drops, and levels
of CD4 cells rebound. However, the immune response is unable to clear the infection completely, and
HIV-infected cells persist in the lymph nodes.
• This period of clinical latency may last for 10 years or more. During this time, there is a high level of
ongoing viral replication.
• It is estimated that 10 billion HIV particles are produced and destroyed each day.
• The half-life of the virus in plasma is about 6 hours, and the virus life cycle (from the time of infection of
a cell to the production of new progeny that infect the next cell) averages 2.6 days. CD4 T lymphocytes,
major targets responsible for virus production, appear to have similar high turnover rates.
• Once productively infected, the half-life of a CD4 lymphocyte is about 1.6 days.

172. A 10-month-old infant who was born 4 weeks premature was brought to the ED with high fever,
rhinorrhea, cough, and difficulty breathing. On examination, the baby had dyspnea and tachypnea;
rales and wheezing were heard over both lungs. The baby was admitted to PICU where she suffered
respiratory failure and was placed on mechanical ventilation. Two different types of tests for
respiratory syncytial virus (RSV or Pneumovirus) were negative, as were tests for parainfluenza virus,
influenza A and B viruses, and adenovirus. Which of the following viruses is the most likely etiologic
agent?
a. Cytomegalovirus
b. Human metapneumovirus
c. Parvovirus B19
d. Rhinovirus

Ans: B
Ref: Levinson, Ch 39, 44, 46. Murray, Ch 56. Ryan, Ch 9, 10.)
The most likely etiologic agent of this infant’s pneumonia is the human metapneumovirus, a member of the
Paramyxoviridae closely related to but distinct from respiratory syncytial virus. Discovered in 2001, this virus is
now known to be responsible for 5% to 15% of serious lower respiratory tract infections in young children. As
with RSV, infants born prematurely and those with preexisting lung or heart disease or immunocompromising are
at greater risk for serious infection with hMPV. Cytomegalovirus and HSV type 1 may cause pneumonia in
immunocompromised infants, but the presentation would be different. Parvovirus B19 does not cause pneumonia,
and only occasional strains of rhinovirus may cause pneumonia as most strains prefer to grow at the much lower
temperatures found in the nasal passages.

173. What is the vector of Trypanosoma cruzi?


a. Bed bug
b. Kissing bug
c. Sand fly
d. Tsetse fly

Ans: B, Kissing bug


Ref: Jawetz Microbiology, 27th edition, page 714

Paradise Institute & Self Study Centre Page 75


PARADISE WEEKLY MODEL TEST – CEE MD/MS BASED QAE SHRAWAN 20

174. What is the first stain used during Gram staining?


a. Carbol fuchsin
b. Crystal violet
c. Methylene blue
d. Safranin

Ans: B, Crystal violet


Ref: Jawetz Microbiology, 27th edition, page 743
Gram stain:
• Fix smear using heat or methanol.
• Cover with crystal violet stain (10–30 seconds).
• Rinse with water. Do not blot.
• Counterstain with Gram’s iodine stain (10–30 seconds).
• Rinse with water. Do not blot.
• Decolorize with gentle agitation in 30% acetone-alcohol (10–30 seconds, until stain no longer flows off
slide).
• Rinse with water. Do not blot.
• Cover with safranin stain (10–30 seconds).
• Rinse with water and air or blot dry.

175. Which of the following immune cells are also known as large granular lymphocytes?
a. B cells
b. T cells
c. Null cells
d. Neutrophils

Ans: C
Ref: Ananthanarayan Microbiology, 10th edition, page 139
• When circulating lymphocytes are classified by their surface markers into T and B cells, about 5 – 10 % of the
cells are found to lack features of either type. They are called null cells.
• Because of their morphology, there are also known as large granular lymphocytes (LGL). They are nearly
double the size of small lymphocytes, with indented nuclei and abundant cytoplasm containing several
azurophilic granules, composed of mitochondria, ribosomes, endoplasmic reticulum and Golgi apparatus.
• LGL are a heterogenous group of cells with differences in their functional and surface marker features.
• The most important member of this group is the natural killer (NK) cell. Others are the antibody dependent
cytotoxic cells and the lymphokine activated killer cells. The term NK cell is sometimes used as a common
name for all null cells.

Community Medicine

Paradise Institute & Self Study Centre Page 76


PARADISE WEEKLY MODEL TEST – CEE MD/MS BASED QAE SHRAWAN 20

176. Outdated cytotoxic drugs are best disposed by:


a. Disposal in municipal waste
b. Screw-feed technology
c. Store for months and burial
d. Autoclave

Ans: b
Ref: Park's Textbook of Preventive and Social Medicine; 24th Edition; Page No: 828
• Screw-feed technology is the basis of a non-burn, dry thermal disinfection process in which waste is
shredded and heated in a rotating auger.
• The waste is reduced by 80 per cent in volume and by 20-35 per cent in weight.
• This process is suitable for treating infectious waste and sharps, but it should not be used to process
pathological, cytotoxic or radio-active waste.

177. Incubation period of Rubella is:


a. 2-3 hours
b. 2-3 days
c. 2-3 weeks
d. 2-3 months

Ans: c
Ref: Park's Textbook of Preventive and Social Medicine; 24th Edition; Page No: 160
▪ Rubella or German measles is an acute childhood infection, usually mild, of short duration (approximately
3 days) and accompanied by low- grade fever, lymphadenopathy and a maculopapular rash. Infection in
early pregnancy may result in serious congenital defects, including death of the foetus.
▪ Incubation period: 2 to 3 weeks; average 18 days.

178. Human Developmental index comprise of:


a. Education, occupation and income
b. Education, employment, food and health
c. Infant mortality rare, longevity, literacy
d. Longevity, income & literacy

Ans: d
Ref: Park PSM 23rd Edition, Page No: 17
Human Development Index (HDI)
The Human Development Index (HDI) is a statistic composite index of life expectancy, education, and per capita
income indicators, which are used to rank countries into four tiers of human development.
A country scores a higher HDI when the lifespan is higher, the education level is higher, and the gross national
income GNI (PPP) per capita is higher.

179. Which of the following is not a WHO recommendation for preventive measures against coronary
heart disease?
a. Reduction of fat intake to 20-30 per cent of total energy intake
b. Consumption of saturated fats must be limited to less than 10 per cent of total energy intake
c. Reduction of dietary cholesterol to below 500 mg per 1000 kcal per day an increase in complex
carbohydrate consumption {i.e., vegetables, fruits, whole grains and legumes)
d. Reduction of salt intake to 5 g daily or less

Paradise Institute & Self Study Centre Page 77


PARADISE WEEKLY MODEL TEST – CEE MD/MS BASED QAE SHRAWAN 20

Ans: c
Ref: Park PSM 23rd Edition, Page No: 370
Dietary modification is the principal preventive strategy in the prevention of CHO. The WHO Expert Committee
considered the following dietary changes to be appropriate for high incidence populations:
• Reduction of fat intake to 20-30 per cent of total energy intake
• Consumption of saturated fats must be limited to less than 10 per cent of total energy intake; some of the
reduction in saturated fat may be made up by mono and poly-unsaturated fats
• A reduction of dietary cholesterol to below 100 mg per 1000 kcal per day an increase in complex
carbohydrate consumption {i.e., vegetables, fruits, whole grains and legumes)
• Avoidance of alcohol consumption
• Reduction of salt intake to 5 g daily or less

180. Various milk in order of calcium per 100 gms is:


a. Buffalo > Goat > Cow > Human
b. Buffalo > Cow > Goat > Human
c. Human > Buffalo > Goat > Cow
d. Goat > Cow > Buffalo > Human

Ans: a
Ref: Park PSM 23rd Edition, Page No: 630

181. Cradle of civilization is


a. Mesopotamia
b. Harappa
c. Mohenjadero
d. Sindhu ghati

Ans: a
Ref: Park PSM 23rd Edition, Page No: 3
Mesopotamian medicine
Contemporary with ancient Egyptian civilization, there existed another civilization in the land which lies between
the Euphrates and Tigris rivers, Mesopotamia (now part of Iraq), often called the "Cradle of Civilization", as long
as 6,000 years ago.

Research methodology

182. Approach to medical practice intended to optimize decision-making by emphasizing the use of well-
designed and well-conducted research conclusion is:
Paradise Institute & Self Study Centre Page 78
PARADISE WEEKLY MODEL TEST – CEE MD/MS BASED QAE SHRAWAN 20

a. Research based practice


b. Evidence based practice
c. Allopathic practice
d. Homeopathic practice

Ans: b
Evidence-based medicine (EBM) is an approach to medical practice intended to optimize decision-making by
emphasizing the use of evidence from well-designed and well-conducted research.
Although all medicine based on science has some degree of empirical support, EBM goes further, classifying
evidence by its epistemologic strength and requiring that only the strongest types (coming from meta-analyses,
systematic reviews, and randomized controlled trials) can yield strong recommendations; weaker types (such as
from case-control studies) can yield only weak recommendations

183. Which of the following denotes measures of variability?


a. Range
b. Mean deviation
c. Standard deviation
d. All of the above

Ans: d
Measures of Variability
• Range
• Inter-quartile range
• Mean deviation
• Standard deviation
• Coefficient of variation

184. In study first schools are sampled, then sections, and finally students. This type of sampling is known
as:
a. Stratified sampling
b. Simple random sampling
c. Cluster sampling
d. Multistage sampling

Ans: d
Ref: Mahajan Biostatistics 8th Edition, page No: 124
Multistage Sampling
▪ As the name implies, this method refers to the sampling procedures carried out several stages using
random sampling techniques.
▪ This is employed in large country surveys. In the first stage, random numbers of districts are chosen in all
the states, followed by random numbers of talukas, villages and units, respectively, e.g. for hookworm
survey in a district, choose 10% villages in the talukas and then examine stools of all persons in every
10th house.
Merits
▪ It introduces flexibility in sampling, which is lacking in other techniques.
It enables the use of existing division and subdivision which saves extra labor.

185. Declaration of Helsinki is about:


a. Human Research ethics

Paradise Institute & Self Study Centre Page 79


PARADISE WEEKLY MODEL TEST – CEE MD/MS BASED QAE SHRAWAN 20

b. e-support use in Biostatistics


c. Formation of at least 1 research centre in every country
d. Fund support to medically deprived countries
Ans: a
Ref: World Medical Association (2013). "Declaration of Helsinki: Ethical Principles for Medical Research
Involving Human Subjects". JAMA. 310 (20): 2191–2194
The Declaration of Helsinki is a set of ethical principles regarding human experimentation developed for the
medical community by the World Medical Association (WMA). It is widely regarded as the cornerstone
document on human research ethics.

Forensic Medicine

186. A 4th year medical student go accidental needlestick injury. Later on investigation the patient was
found to be HIV positive. Transmission rate of HIV in such case is:
a. 0.3%
b. 3%
c. 5%
d. 10%

Ans: a
Ref: Gautam Biswas Forensic Medicine, 3rdEdition, Page No: 123
Risk of transmission from single percutaneous exposure to blood for:
▪ HBV: 6–30%
▪ HIV: 0.1–0.36%
▪ HCV: 2.7–10%

187. Lichtenberg flowers are seen in:


a. Thermal burns
b. Electrocution
c. Vitriolage
d. Lightning

Ans: d
Ref: Gautam Biswas Forensic Medicine, 3rdEdition, Page No: 278
▪ Lichtenberg flowers: It is known by different names like ‘arborization’, ‘feathering’, ‘ferning’, ‘filigree
burns’ or ‘keraunographic markings’.
▪ Lichtenberg flowers are pathognomonic of lightning strike, but are rare.
▪ These are superficial, several inches long, thin, irregular, tortuous, dendritic red marks on the skin.
▪ These marks have a resemblance to the branches of a tree.
▪ This fern-like pattern of erythema in the skin is usually found over the shoulders or flanks.
▪ It is not associated with burning.
▪ They indicate the path taken by the discharge and tend to follow skin creases and the long axis of the
body.
▪ It appears within 1 h, and disappears in 24–48 h, if the person survive

188. Universal antidote contains the following, except:


a. Powdered charcoal
b. Tannic acid
c. Ground mustard

Paradise Institute & Self Study Centre Page 80


PARADISE WEEKLY MODEL TEST – CEE MD/MS BASED QAE SHRAWAN 20

d. Magnesium oxide

Ans: c
Ref: Gautam Biswas Forensic Medicine, 3rdEdition, Page No: 475
Universal Antidote: It is a combination of physical and chemical antidotes; used in those cases where the nature
of ingested poisons is unknown or when it is suspected that two or more poisons were taken.
▪ Powdered charcoal (burnt toast): 2 parts: Adsorbs alkaloids
▪ Magnesium oxide (milk of magnesia): 1 part: Neutralizes acids
▪ Tannic acid (strong tea): 1 part: Precipitates alkaloids, glycosides and metals

189. Risk of thromboembolism is highest with:


a. Deep femoral vein thrombus
b. Anterior tibial vein thrombus
c. Posterior tibial vein thrombus
d. Popliteal vein thrombus

Ans: a
Ref: Gautam Biswas Forensic Medicine, 3rdEdition, Page No: 301
Thrombosis and embolism (thromboembolism)
It is a common complication of traumatic lesions of lower extremities.
Most common sites of thrombosis are: Deep femoral, popliteal and posterior tibial veins.
Factors which predispose to leg vein thrombosis after injury are:
• Local tissue damage, causing injury to veins.
• An increase in clotting time, which is maximum at about 2 weeks after injury.
• Immobility and bed rest.
• General debility, especially in old age, leading to poor general circulation and cardiac output.
Thrombus usually develops in 10–20 days after injury, gets detached in part or whole and can cause pulmonary
embolism (saddle embolism).

190. Most accurate method of diagnosis of pregnancy at 6 weeks:


a. Hegar’s sign
b. X-ray examination
c. Palpation of fetal parts
d. Fetal heart sound by USG

Ans: d
Ref: Gautam Biswas Forensic Medicine, 3rdEdition, Page No: 359

Paradise Institute & Self Study Centre Page 81


PARADISE WEEKLY MODEL TEST – CEE MD/MS BASED QAE SHRAWAN 20

Mandatory CPD Topics

191. For an informed consent to be valid, the patient has to be given:


a. Potential benefits and risks, side effects and harms of available treatment options
b. Their alternatives, their consequences and complications
c. The consequences of no treatment which enable the patients to exercise their right to self-decision
d. All of the above

Ans: d
Informed Consent
Informed consent is the fundamental ethical and legal cornerstone of the doctor-patient relationship. For an
informed consent to be valid, the patient has to be given
▪ Adequate information regarding nature of illness,
▪ Potential benefits and risks, side effects and harms of available treatment options,
▪ Their alternatives, their consequences and complications, and
▪ The consequences of no treatment which enable the patients to exercise their right to self-decision

192. A medical doctor should follow which of the following while exercising organ transplantation?
a. Human Organ Transplant Act 2051 amended on 2072
b. Human Organ Transplant Act 2053 amended on 2074
c. Human Organ Transplant Act 2055 amended on 2074
d. Human Organ Transplant Act 2055 amended on 2072

Ans: d
Organ transplant and organ donation
Medical practitioners should exercise the principles and the provisions of the Human Organ Transplant Act 2055
amended on 2072

193. Which of the following is correct recommendation when you choose to wear a mask?
a. Before touching the mask, clean hands with an alcohol-based hand rub or soap and water
b. Take the mask and inspect it for tears or holes
c. Place the mask to your face. Pinch the metal strip or stiff edge of the mask so it moulds to the shape of
your nose
d. All of the above

Ans: d
If you choose to wear a mask:
▪ Before touching the mask, clean hands with an alcohol-based hand rub or soap and water
▪ Take the mask and inspect it for tears or holes.
▪ Orient which side is the top side (where the metal strip is).
▪ Ensure the proper side of the mask faces outwards (the colored side).
▪ Place the mask to your face. Pinch the metal strip or stiff edge of the mask so it moulds to the shape of
your nose.
▪ Pull down the mask’s bottom so it covers your mouth and your chin.
▪ Do not touch the mask while you are wearing it for protection.
▪ After use, take off the mask with clean hands; remove the elastic loops from behind the ears while keeping
the mask away from your face and clothes, to avoid touching potentially contaminated surfaces of the
mask.

Paradise Institute & Self Study Centre Page 82


PARADISE WEEKLY MODEL TEST – CEE MD/MS BASED QAE SHRAWAN 20

▪ Discard the mask in a closed bin immediately after use. Do not reuse the mask.
▪ Perform hand hygiene after touching or discarding the mask – Use alcohol-based hand rub or, if visibly
soiled, wash your hands with soap and water.

194. Bag-valve-mask is
a. Best option for prehospital airway support.
b. Appropriate for elective ventilation in the operating room
c. Emergency skill
d. All of the above

Ans: d
▪ Bag-valve-mask (BVM) ventilation is an essential emergency skill. This basic airway management
technique allows for oxygenation and ventilation of patients until a more definitive airway can be
established and in cases where endotracheal intubation or other definitive control of the airway is not
possible. For the emergency medical technician, basic BVM ventilation is most often the only option for
airway management.
▪ In the pediatric population, BVM may be the best option for prehospital airway support. BVM ventilation
is also appropriate for elective ventilation in the operating room (OR) when intubation is not required, but
it is now often replaced in this setting by the laryngeal mask airway

195. In a PHCC, a person was regular on tubercular medication for 2 weeks. He left taking medicine for 1
month and came again. What will you do?
a. Scold him for his irregular behaviour
b. Try to find out why he missed the month
c. Contact trace the visits of this patient
d. Ask him to show his drug chart

Ans: b
In cases of non-compliant patients, always try to seek why the patient is not adhering to the drugs. Many social
factors and drug reactions should be studied.

196.Which of the following is not social influencing skill?


a. Interpretation/reframing
b. Challenge/directive
c. Self-disclosure
d. Entertainment

Ans: d
Social influencing skills help clients to explore more deeply their concerns and encourage them to make changes
in their attitudes and behaviors. Some of the most important social influencing skills are:
- Interpretation/reframing: provide client with a new way to view or understand the situation
- Challenge/directive: support clients but pointing out discrepancies or mixed messages in their thoughts or
behavior and suggesting alternative ways of thinking and behaving
- Self-disclosure: clinicians share selective personal experiences with the client when appropriate
- Feedback: provides clients with information on how others might perceive their thoughts or behavior
- Influencing summary: clarifies what has been discussed so client will be encouraged to think or act in
different ways between sessions

Paradise Institute & Self Study Centre Page 83


PARADISE WEEKLY MODEL TEST – CEE MD/MS BASED QAE SHRAWAN 20

197. During communication, repeating back to patients what they have said to encourage them to elaborate
is:
a. Encouraging
b. Paraphrasing
c. Reflection of feeling
d. Summarization

Ans: a
Good attending skills communicate the clinician’s undivided attention to the client’s concerns. Attending
behavior encourages the client to freely talk and therefore reduces the need for the clinician to talk. Attending is
both an attitude and a skill that requires paying attention to and practicing. Some of the basic listening and
attending skills are:
▪ Open questions: what, when, how
▪ Closed questions: usually begin with “do,” “is,” or “are,” and the question can usually be answered in a
few words
▪ Encouraging: repeating back to clients what they have said to encourage them to elaborate
▪ Paraphrasing: repeating back what the client is saying to show understanding and encourage elaboration
by the client
▪ Reflection of feeling: attention to the emotional content of what the person is saying, doing, and feeling
▪ Summarization: to clarify what has been discussed so far

198. Which of the following is wrong?


a. 30 Compressions for every 2 breaths given during one rescuer CPR for a child
b. 30 Compressions for every 2 breaths given during one rescuer CPR for an infant
c. 30 Compressions for every 2 breaths given during two rescuer CPR for a child
d. An adult’s chest should be pushed down 2” during compressions

Ans: c
A lone rescuer uses a compression-to-ventilation ratio of 30:2. For 2-rescuer infant and child CPR, one provider
should perform chest compressions while the other keeps the airway open and performs ventilations at a ratio of
15:2
The key numbers to know for a CPR exam based on the 2010 American Heart Association standards are:
▪ 30 Compressions for every 2 breaths given during one rescuer CPR for a child
▪ 30 Compressions for every 2 breaths given during one rescuer CPR for an infant
▪ 15 Compressions for every 2 breaths given during two rescuer CPR for a child
▪ 15 Compressions for every 2 breaths given during two rescuer CPR for an infant
▪ Compressions must be given at a rate between 100 and 120 per minute
▪ An adults chest should be pushed down 2” during compressions
▪ Breaths should be given over 1 second

199. The most common means of spreading infection are:


a. Soiled instruments
b. Infected patients
c. Human hands
d. Domestic animals

Ans: c
Hand washing is the single most effective way to prevent the spread of infections.
An infection occurs when germs enter the body, increase in number, and cause a reaction of the body.

Paradise Institute & Self Study Centre Page 84


PARADISE WEEKLY MODEL TEST – CEE MD/MS BASED QAE SHRAWAN 20

Three things are necessary for an infection to occur:


▪ Source: Places where infectious agents (germs) live (e.g., sinks, surfaces, human skin)
▪ Susceptible Person with a way for germs to enter the body
▪ Transmission: a way germs are moved to the susceptible persons

200. Cardiac output generated through CPR during cardiac arrest is roughly:
a. 5-10 percent
b. 10-20 percent
c. 20-30 percent
d. 40-50 percent
Ans: c
Ref: Update on cardiopulmonary resuscitation guidelines of interest to anesthesiologists; Luiz Fernando
dos Reis Falcão; David Ferez; José Luiz Gomesdo Amaral
During CPR the cardiac output is approximately 25% to 33% of normal; thus, oxygen and CO2 exchange by the
lungs is reduced. As a consequence, low minute volume (lower than the normal tidal volume and respiratory rate)
can maintain effective oxygenation and ventilation.

Paradise Institute & Self Study Centre Page 85

You might also like